Exam 2 Review Questions- CV and HTN, Respiratory, Ostomy Flashcards

1
Q

Which of the following terms is used to describe the ability of the heart to initiate an electrical impulse?

a) Automaticity
b) Excitability
c) Contractility
d) Conductivity

A

Automaticity

Automaticity is the ability of specialized electrical cells of the cardiac conduction system to initiate an electrical impulse.

Contractility refers to the ability of the specialized electrical cells of the cardiac conduction system to contract in response to an electrical impulse.

Conductivity refers to the ability of the specialized electrical cells of the cardiac conduction system to transmit an electrical impulse from one cell to another.

Excitability refers to the ability of the specialized electrical cells of the cardiac conduction system to respond to an electrical impulse.

How well did you know this?
1
Not at all
2
3
4
5
Perfectly
2
Q

Age-related changes associated with the cardiac system include which of the following?

Select all that apply.

a) Increase in the number of SA node cells
b) Myocardial thinning
c) Endocardial fibrosis
d) Increased size of the left atrium

A

Endocardial fibrosis Increased size of the left atrium

Age-related changes associated with the cardiac system include endocardial fibrosis, increased size of the left atrium, decreased number of SA node cells, and myocardial thickening.

How well did you know this?
1
Not at all
2
3
4
5
Perfectly
3
Q

For both outpatients and inpatients scheduled for diagnostic procedures of the cardiovascular system, the nurse performs a thorough initial assessment to establish accurate baseline data. Which of the following data is necessary to collect if the patient is experiencing chest pain?

a) Blood pressure in the left arm
b) Description of the pain
c) Sound of the apical pulses
d) Pulse rate in upper extremities

A

Description of the pain

If the patient is experiencing chest pain, a history of its location, frequency, and duration is necessary, as is a description of the pain, if it radiates to a particular area, what precipitates its onset, and what brings relief.

The nurse weighs the patient and measures vital signs. The nurse may measure BP in both arms and compare findings.

The nurse assesses apical and radial pulses, noting rate, quality, and rhythm. The nurse also checks peripheral pulses in the lower extremities.

How well did you know this?
1
Not at all
2
3
4
5
Perfectly
4
Q

The nurse is caring for a patient prescribed warfarin (Coumadin) orally. The nurse reviews the patient’s prothrombin time (PT) level to evaluate the effectiveness of the medication. The nurse should also evaluate which of the following laboratory values?

a) Partial thromboplastic time (PTT)
b) Complete blood count (CBC)
c) Sodium
d) International normalized ratio (INR)

A

International normalized ratio (INR)

The INR, reported with the PT, provides a standard method for reporting PT levels and eliminates the variation of PT results from different laboratories.

The INR, rather than the PT alone, is used to monitor the effectiveness of warfarin.

The therapeutic range for INR is 2 to 3.5, although specific ranges vary based on diagnosis.

The other laboratory values are not used to evaluate the effectiveness of Coumadin.

How well did you know this?
1
Not at all
2
3
4
5
Perfectly
5
Q

The nurse is caring for a patient in the ICU diagnosed with coronary artery disease (CAD). Which of the following assessment data indicates the patient is experiencing a decrease in cardiac output?

a) BP 108/60 mm Hg, ascites, and crackles
b) Disorientation, 20 mL of urine over the last 2 hours
c) Reduced pulse pressure and heart murmur
d) Elevated jugular venous distention (JVD) and postural changes in BP

A

Disorientation, 20 mL of urine over the last 2 hours

Assessment findings associated with reduced cardiac output include reduced pulse pressure, hypotension, tachycardia, reduced urine output, lethargy, or disorientation.

How well did you know this?
1
Not at all
2
3
4
5
Perfectly
6
Q

During the auscultation of a patient’s heart sounds, the nurse notes an S4. The nurse recognizes that an S4 is associated with which of the following?

a) Turbulent blood flow
b) Heart failure
c) Hypertensive heart disease
d) Diseased heart valves

A

Hypertensive heart disease

Auscultation of the heart requires familiarization with normal and abnormal heart sounds.

An extra sound just before S1 is an S4 heart sound, or atrial gallop.

An S4 sound often is associated with hypertensive heart disease.

A sound that follows S1 and S2 is called an S3 heart sound or a ventricular gallop.

An S3 heart sound is often an indication of heart failure in an adult.

In addition to heart sounds, auscultation may reveal other abnormal sounds, such as murmurs and clicks, caused by turbulent blood flow through diseased heart valves.

How well did you know this?
1
Not at all
2
3
4
5
Perfectly
7
Q

The nurse is screening a patient prior to a magnetic resonance angiogram (MRA) of the heart. Which of the following actions should the nurse complete prior to the patient undergoing the procedure? Select all that apply.

a) Sedate the patient prior to the procedure.
b) Remove the patient’s Transderm Nitro patch.
c) Offer the patient a headset to listen to music during the procedure.
d) Remove the patient’s jewelry.
e) Position the patient on his/her stomach for the procedure.

A

Remove the patient’s Transderm Nitro patch.

Offer the patient a headset to listen to music during the procedure.

Remove the patient’s jewelry.

Transdermal patches that contain a heat-conducting aluminized layer (e.g., NicoDerm, Androderm, Transderm Nitro, Transderm Scop, Catapres-TTS) must be removed before MRA to prevent burning of the skin.

A patient who is claustrophobic may need to receive a mild sedative before undergoing an MRA.

During an MRA, the patient is positioned supine on a table that is placed into an enclosed imager or tube containing the magnetic field.

Patients are instructed to remove any jewelry, watches, or other metal items (e.g., ECG leads).

An intermittent clanking or thumping that can be annoying is generated by the magnetic coils, so the patient may be offered a headset to listen to music.

How well did you know this?
1
Not at all
2
3
4
5
Perfectly
8
Q

The nurse is caring for a patient in the ED who has a B-type natriuretic peptide (BNP) level of 115 pg/mL. The nurse understands that this finding is most suggestive of which of the following?

a) Heart failure
b) Myocardial infarction
c) Ventricular hypertrophy
d) Pulmonary edema

A

Heart failure

A BNP level greater than 100 pg/mL is suggestive of HF. Because this serum laboratory test can be quickly obtained, BNP levels are useful for prompt diagnosis of HF in settings such as the ED.

Elevations in BNP can occur from a number of other conditions such as pulmonary embolus, myocardial infarction (MI), and ventricular hypertrophy.

Therefore, the clinician correlates BNP levels with abnormal physical assessment findings and other diagnostic tests before making a definitive diagnosis of HF.

How well did you know this?
1
Not at all
2
3
4
5
Perfectly
9
Q

The nurse is caring for a patient with an intra-arterial BP monitoring device. The nurse recognizes the most preventable complication associated with hemodynamic monitoring includes which of the following?

a) Catheter-related bloodstream infections (CRBSI)
b) Pneumothorax
c) Hemorrhage
d) Air embolism

A

Catheter-related bloodstream infections (CRBSI)

CRBSIs are the most common preventable complication associated with hemodynamic monitoring systems.

Comprehensive guidelines for the prevention of these infections have been published by Centers for Disease Control and Prevention (CDC).

Complications from use of hemodynamic monitoring systems are uncommon and can include pneumothorax, infection, and air embolism.

A pneumothorax may occur during the insertion of catheters using a central venous approach (CVP and pulmonary artery catheters).

Air emboli can be introduced into the vascular system if the stopcocks attached to the pressure transducers are mishandled during blood drawing, administration of medications, or other procedures that require opening the system to air.

How well did you know this?
1
Not at all
2
3
4
5
Perfectly
10
Q

The nurse is observing a patient during an exercise stress test (bicycle). Which of the following findings indicates a positive test and the need for further diagnostic testing?

a) ST-segment changes on the ECG
b) BP changes; 148/80 mm Hg to 166/90 mm Hg
c) Dizziness and leg cramping
d) Heart rate changes; 78 bpm to 112 bpm

A

ST-segment changes on the ECG

During the test, the following are monitored: two or more ECG leads for heart rate, rhythm, and ischemic changes; BP; skin temperature; physical appearance; perceived exertion; and symptoms, including chest pain, dyspnea, dizziness, leg cramping, and fatigue.

The test is terminated when the target heart rate is achieved or if the patient experiences signs of myocardial ischemia.

Further diagnostic testing, such as a cardiac catheterization, may be warranted if the patient develops chest pain, extreme fatigue, a decrease in BP or pulse rate, serious dysrhythmias or ST-segment changes on the ECG during the stress test. The other findings would not warrant the testing to be stopped.

How well did you know this?
1
Not at all
2
3
4
5
Perfectly
11
Q

The nurse is reviewing discharge instructions with a patient who underwent a left groin cardiac catheterization 8 hours ago. Which of the following instructions should the nurse include?

a) “Contact your primary care provider if you develop a temperature above 102°F.”
b) “Do not bend at the waist, strain, or lift heavy objects for the next 24 hours.”
c) “If any discharge occurs at the puncture site, call 911 immediately.”
d) “You can take a tub bath or a shower when you get home.”

A

“Do not bend at the waist, strain, or lift heavy objects for the next 24 hours.”

The nurse should instruct the patient to complete the following:

If the artery of the groin was used, for the next 24 hours, do not bend at the waist, strain, or lift heavy objects; the primary provider should be contacted if any of the following occur: swelling, new bruising or pain from your procedure puncture site, temperature of 101°F or more. If bleeding occurs, lie down (groin approach) and apply firm pressure to the puncture site for 10 minutes.

Notify the primary provider as soon as possible and follow instructions. If there is a large amount of bleeding, call 911. The patient should not drive to the hospital.

How well did you know this?
1
Not at all
2
3
4
5
Perfectly
12
Q

The nurse is caring for a patient with clubbing of the fingers and toes. The nurse should complete which of the following actions given these findings?

a) Obtain an oxygen saturation level.
b) Assess the patient for pitting edema.
c) Obtain a 12-lead ECG tracing.
d) Assess the patient’s capillary refill.

A

Obtain an oxygen saturation level.

Clubbing of the fingers and toes indicates chronic hemoglobin desaturation (decreased oxygen supply) and is associated with congenital heart disease.

The nurse should assess the patient’s O2 saturation level and intervene as directed. The other assessments are not indicated.

How well did you know this?
1
Not at all
2
3
4
5
Perfectly
13
Q

The nurse auscultates the PMI (point of maximal impulse) at which of the following anatomic locations?

a) Left midclavicular line, fifth intercostal space
b) 2 inches to the left of the lower end of the sternum
c) 1 inch to the left of the xiphoid process
d) Midsternum

A

Left midclavicular line, fifth intercostal space

The left ventricle is responsible for the apical impulse or the point of maximum impulse, which is normally palpable in the left midclavicular line of the chest wall at the fifth intercostal space.

The right ventricle lies anteriorly, just beneath the sternum. Use of inches to identify the location of the PMI is inappropriate based on variations in human anatomy.

Auscultation below and to the left of the xiphoid process will detect gastrointestinal sounds, but not the PMI.

How well did you know this?
1
Not at all
2
3
4
5
Perfectly
14
Q

When the balloon on the distal tip of a pulmonary artery catheter is inflated and a pressure is measured, the measurement obtained is referred to as which of the following?

a) Central venous pressure
b) Pulmonary artery wedge pressure
c) Cardiac output
d) Pulmonary artery pressure

A

Pulmonary artery wedge pressure

When the balloon is inflated, the tip of the catheter floats into smaller branches of the pulmonary artery until it can no longer be passed and the pressure is recorded, reflecting left atrial pressure and left ventricular end-diastolic pressure.

Central venous pressure is measured in the right atrium. Pulmonary artery pressure is measured when the balloon tip is not inflated.

Cardiac output is determined through thermodilution involving injection of fluid into the pulmonary artery catheter.

How well did you know this?
1
Not at all
2
3
4
5
Perfectly
15
Q

A nurse is preparing to assess a patient for postural BP changes. Which of the following indicates the need for further education?

a) Obtaining the supine measurements prior to the sitting and standing measurements
b) Taking the patient’s BP with the patient sitting on the edge of the bed with feet dangling
c) Positioning the patient supine for 10 minutes prior to taking the initial BP and HR
d) Letting 30 seconds elapse after each position change before measuring BP and heart rate (HR)

A

Letting 30 seconds elapse after each position change before measuring BP and heart rate (HR)

The following steps are recommended when assessing patients for postural hypotension:

Position the patient supine for 10 minutes before taking the initial BP and HR measurements; reposition the patient to a sitting position with legs in the dependent

position, wait 2 minutes then reassess both BP and HR measurements; if the patient is symptom free or has no significant decreases in systolic or diastolic BP, assist the patient into a standing position, obtain

measurements immediately and recheck in 2 minutes; continue measurements every 2 minutes for a total of 10 minutes to rule out postural hypotension.

Return the patient to supine position if postural hypotension is detected or if the patient becomes symptomatic.

Document HR and BP measured in each position (e.g., supine, sitting, and standing) and any signs or symptoms that accompany the postural changes.

How well did you know this?
1
Not at all
2
3
4
5
Perfectly
16
Q

The ability of the cardiac muscle to shorten in response to an electrical impulse is termed which of the following?

a) Contractility
b) Diastole
c) Depolarization
d) Repolarization

A

Contractility

Contractility is the ability of the cardiac muscle to shorten in response to an electrical impulse.

Depolarization is the electrical activation of a cell caused by the influx of sodium into the cell while potassium exits the cell.

Repolarization is the return of the cell to the resting state, caused by reentry of potassium into the cell while sodium exits the cell.

Diastole is the period of ventricular relaxation resulting in ventricular filling.

How well did you know this?
1
Not at all
2
3
4
5
Perfectly
17
Q

The nurse is caring for a patient who has undergone peripheral arteriography. How should the nurse assess the adequacy of peripheral circulation?

a) By hemodynamic monitoring
b) By checking peripheral pulses
c) By observing the patient for bleeding
d) By checking for cardiac dysrhythmias

A

By checking peripheral pulses

Peripheral arteriography is used to diagnose occlusive arterial disease in smaller arteries.

The nurse observes the patient for bleeding and cardiac dysrhythmias and assesses the adequacy of peripheral circulation by frequently checking the peripheral pulses.

Hemodynamic monitoring is used to assess the volume and pressure of blood in the heart and vascular system.

How well did you know this?
1
Not at all
2
3
4
5
Perfectly
18
Q

Decreased pulse pressure reflects which of the following?

a) Tachycardia
b) Reduced stroke volume
c) Elevated stroke volume
d) Reduced distensibility of the arteries

A

Reduced stroke volume

Decreased pulse pressure reflects reduced stroke volume and ejection velocity or obstruction to blood flow during systole.

Increased pulse pressure would indicate reduced distensibility of the arteries, along with bradycardia.

How well did you know this?
1
Not at all
2
3
4
5
Perfectly
19
Q

Which of the following is the term for the normal pacemaker of the heart?

a) Sinoatrial (SA) node
b) Atrioventricular (AV) node
c) Purkinje fibers
d) Bundle of His

A

Sinoatrial (SA) node

The sinoatrial node is the primary pacemaker of the heart.

The AV node coordinates the incoming electrical impulses from the atria and, after a slight delay, relays the impulse to the ventricles.

The Purkinje fibers rapidly conduct the impulses through the thick walls of the ventricles.

How well did you know this?
1
Not at all
2
3
4
5
Perfectly
20
Q

The nurse correctly identifies which of the following data as an example of BP and HR measurements in a patient with postural hypotension?

a) Supine: BP 114/82 mm Hg, HR 90 bpm; sitting: BP 110/76 mm Hg, HR 95 bpm; standing: BP 108/74 mm Hg, HR 98 bpm
b) Supine: BP 140/78 mm Hg, HR 72 bpm; sitting: BP 145/78 mm Hg, HR 74 bpm; standing: BP 144/78 mm Hg, HR 74 bpm
c) Supine: BP 130/70 mm Hg, HR 80 bpm; sitting: BP 128/70 mm Hg, HR 80 bpm; standing: BP 130/68 mm Hg, HR 82 bpm
d) Supine: BP 120/70 mm Hg, HR 70 bpm; sitting: BP 100/55 mm Hg, HR 90 bpm; standing: BP 98/52 mm Hg, HR 94 bpm

A

Supine: BP 120/70 mm Hg, HR 70 bpm; sitting: BP 100/55 mm Hg, HR 90 bpm; standing: BP 98/52 mm Hg, HR 94 bpm

Postural (orthostatic) hypotension is a sustained decrease of at least 20 mm Hg in systolic BP or 10 mm Hg in diastolic BP within 3 minutes of moving from a lying or sitting to a standing position. The following is an example of BP and HR measurements in a patient with postural hypotension: supine:

BP 120/70 mm Hg, HR 70 bpm; sitting: BP 100/55 mm Hg, HR 90 bpm; standing: BP 98/52 mm Hg, HR 94 bpm. Normal postural responses that occur when a person moves from a lying to a standing position include

(1) a HR increase of 5 to 20 bpm above the resting rate;
(2) an unchanged systolic pressure, or a slight decrease of up to 10 mm Hg; and (3) a slight increase of 5 mm Hg in diastolic pressure.

How well did you know this?
1
Not at all
2
3
4
5
Perfectly
21
Q

The area of the heart that is located at the third intercostal (IC) space to the left of the sternum is which of the following?

a) Epigastric area
b) Aortic area
c) Pulmonic area
d) Erb’s point

A

Erb’s point

Erb’s point is located at the third IC space to the left of the sternum.

The aortic area is located at the second IC space to the right of the sternum.

The pulmonic area is at the second IC space to the left of the sternum.

The epigastric area is located below the xiphoid process.

How well did you know this?
1
Not at all
2
3
4
5
Perfectly
22
Q

Which of the following findings indicates that hypertension is progressing to target organ damage?

a) Urine output of 60 cc/mL over 2 hours
b) Blood urea nitrogen (BUN) level of 12 mg/dL
c) Chest x-ray showing pneumonia
d) Retinal blood vessel damage

A

Retinal blood vessel damage

Symptoms suggesting that hypertension is progressing to the extent that target organ damage is occurring must be detected early so that appropriate treatment can be initiated.

All body systems must be assessed to detect any evidence of vascular damage.

An eye examination with an ophthalmoscope is important because retinal blood vessel damage indicates similar damage elsewhere in the vascular system.

The patient is questioned about blurred vision, spots in front of the eyes, and diminished visual acuity.

The heart, nervous system, and kidneys are also carefully assessed.

A BUN level and 60 cc/mL over 2 hours are normal findings.

The presence of pneumonia does not indicate target organ damage.

How well did you know this?
1
Not at all
2
3
4
5
Perfectly
23
Q

A 35-year-old female patient has been diagnosed with hypertension. The patient is a stock broker, smokes daily, and is also a diabetic. During a follow-up appointment, the patient states that she finds it cumbersome and time consuming to visit the doctor regularly just to check her blood pressure (BP). As the nurse, which of the following aspects of patient teaching would you recommend?

a) Advising a smoking cessation
b) Administering glycemic control
c) Purchasing a self-monitoring BP cuff
d) Discussing methods for stress reduction

A

Purchasing a self-monitoring BP cuff

Because this patient finds it time consuming to visit the doctor just for a blood pressure reading, as the nurse, you can suggest the use of an automatic cuff at a local pharmacy, or purchasing a self-monitoring cuff.

Discussing methods for stress reduction, advising a smoking cessation, and administering glycemic control would constitute patient education in managing hypertension.

How well did you know this?
1
Not at all
2
3
4
5
Perfectly
24
Q

Officially, hypertension is diagnosed when the patient demonstrates a systolic blood pressure greater than ______ mm Hg and a diastolic blood pressure greater than _____ mm Hg over a sustained period.

a) 120, 70
b) 140, 90
c) 110, 60
d) 130, 80

A

140, 90

According to the categories of blood pressure levels established by the Joint National Committee (JNC) VI, stage 1 hypertension is demonstrated by a systolic pressure of 140 to 159, or a diastolic pressure of 90 to 99. Pressure of 130 systolic and 80 diastolic falls within the normal range for an adult.

Pressure of 110 systolic and 60 diastolic falls within the normal range for an adult.

Pressure of 120 systolic and 70 diastolic falls within the normal range for an adult.

How well did you know this?
1
Not at all
2
3
4
5
Perfectly
25
Q

The nurse teaches the patient which of the following guidelines regarding lifestyle modifications for hypertension?

a) Reduce smoking to no more than four cigarettes per day
b) Stop alcohol intake
c) Limit aerobic physical activity to 15 minutes, three times per week
d) Maintain adequate dietary intake of fruits and vegetables

A

Maintain adequate dietary intake of fruits and vegetables

Guidelines include adopting the dietary approaches to stop hypertension (DASH) eating plan:

consume a diet rich in fruits, vegetables, and low-fat dairy products with a reduced content of saturated and total fat, dietary sodium reduction: reduce dietary sodium intake to no more than 100 mmol/day (2.4 g sodium or 6 g sodium chloride),

and physical activity: engage in regular aerobic physical activity such as brisk walking (at least 30 min/day, most days of the week),

Moderate alcohol consumption: limit consumption to no more than two drinks (eg, 24 oz beer, 10 oz wine, or 3 oz 80-proof whiskey) per day in most men and to no more than 1 drink per day in women and lighter-weight people.

Tobacco: should be avoided because anyone with high blood pressure is already at increased risk for heart disease, and smoking amplifies this risk.

How well did you know this?
1
Not at all
2
3
4
5
Perfectly
26
Q

A 66-year-old client presents to the emergency room (ER) complaining of a severe headache and mild nausea for the last 6 hours. Upon assessment, the patient’s BP is 210/120 mm Hg. The patient has a history of HTN for which he takes 1.0 mg clonidine (Catapres) twice daily for. Which of the following questions is most important for the nurse to ask the patient next?

a) “Have you taken your prescribed Catapres today?”
b) ”Did you take any medication for your headache?”
c) “Are you having chest pain or shortness of breath?”
d) “Do you have a dry mouth or nasal congestion?”

A

“Have you taken your prescribed Catapres today?”

The nurse must ask if the patient has taken his prescribed Catapres.

Patients need to be informed that rebound hypertension can occur if antihypertensive medications are suddenly stopped.

Specifically, a side effect of Catapres is rebound or withdrawal hypertension. A

lthough the other questions may be asked, it is most important to inquire if the patient has taken his prescribed HTN medication given the patient’s severely elevated BP.

How well did you know this?
1
Not at all
2
3
4
5
Perfectly
27
Q

The nurse is administering medications on a medical surgical unit. A patient is ordered to receive 40 mg of oral Corgard (nadolol) for the treatment of hypertension. Prior to administering the medication, the nurse should complete which of the following?

a) Checking the patient’s heart rate
b) Weighing the patient
c) Checking the patient’s serum K+ level
d) Checking the patient’s urine output

A

Checking the patient’s heart rate

Corgard is a beta-blocker.

A desired effect of this medication is to reduce the pulse rate in patients with tachycardia and an elevated blood pressure (BP).

The nurse should check the patient’s heart rate (HR) prior to administering Corgard to ensure that the patient’s pulse rate is not below 60 (beats per minute (bpm). The other interventions are not indicated prior to administering a beta-blocker medication.

How well did you know this?
1
Not at all
2
3
4
5
Perfectly
28
Q

The nurse is caring for a client who is prescribed diuretic medication for the treatment of hypertension. The nurse recognizes that which of the following medications conserves potassium?

a) Chlorthalidone (Hygroton)
b) Chlorothiazide (Diuril)
c) Furosemide (Lasix)
d) Spironolactone (Aldactone)

A

Spironolactone (Aldactone)

Aldactone is known as a potassium-sparing diuretic. Lasix causes loss of potassium from the body. Diuril causes mild hypokalemia. Hygroton causes mild hypokalemia.

How well did you know this?
1
Not at all
2
3
4
5
Perfectly
29
Q

The nurse is conducting a service project for a local elderly community group on the topic of hypertension. The nurse will relay that risk factors and cardiovascular problems related to hypertension include which of the following? Select all that apply.

a) Elevated high-density lipoprotein (HDL) cholesterol
b) Decreased low-density lipoprotein (LDL) levels.
c) Smoking
d) Age ≥55 in men
e) Obesity (BMI ≥ 30 kg/m2)

A

Smoking

Age ≥55 in men

Obesity (BMI ≥ 30 kg/m2)

Major risk factors (in addition to hypotension) include smoking, dyslipidemia (high LDL, low HDL cholesterol), diabetes mellitus, impaired renal function, obesity, physical inactivity, age (older than 55 years for men, 65 years for women), and family history of cardiovascular disease.

How well did you know this?
1
Not at all
2
3
4
5
Perfectly
30
Q

A 77-year-old woman presents to the local community center for a blood pressure screening. The women’s blood pressure is recorded as 180/90 mm Hg. The woman has a history of hypertension, but she currently is not taking her medications. Which of the following questions is most appropriate for the nurse to ask the patient first?

a) “Are you able to get to your pharmacy to pick up your medications?”
b) “Why is it that you are not taking your medications?”
c) “Are you having trouble paying for your medication?”
d) “What medications are you prescribed?”

A

“Why is it that you are not taking your medications?”

It is important for the nurse to first ascertain if the reason why the patient is not taking her medications.

Adherence to the therapeutic program may be more difficult for older adults.

The medication regimen can be difficult to remember, and the expense can be a challenge. Monotherapy (treatment with a single agent), if appropriate, may simplify the medication regimen and make it less expensive.

The other questions are appropriate, but the priority is to determine why the medication regimen is not being followed.

How well did you know this?
1
Not at all
2
3
4
5
Perfectly
31
Q

The nurse understands that patient education related to antihypertensive medication should include all of the following instructions except which of the following?

a) Avoid over the counter (OTC) cold, weight reduction, and sinus medications.
b) Avoid hot baths, exercise, and alcohol within 3 hours of taking vasodilators.
c) If a dosage of medication is missed, double up on the next one to catch up.
d) Do not stop antihypertensive medication abruptly.

A

If a dosage of medication is missed, double up on the next one to catch up.

Doubling doses could cause serious hypotension (HTN) and is not recommended. Medications should be taken as prescribed.

Hot baths, strenuous exercise, and excessive alcohol are all vasodilators and should be avoided.

Many OTC preparations can precipitate HTN. Stopping antihypertensives abruptly can precipitate a severe hypertensive reaction and is not recommended.

How well did you know this?
1
Not at all
2
3
4
5
Perfectly
32
Q

Target organ damage from untreated/undertreated hypertension includes which of the following? Select all that apply.

a) Diabetes
b) Hyperlipidemia
c) Heart failure

d) Stroke
e) Retinal damage

A

Heart failure

Stroke

Retinal damage

Target organ systems include cardiac, cerebrovascular, peripheral vascular, renal, and the eye. Hyperlipidemia and diabetes are risk factors for development of hypertension.

How well did you know this?
1
Not at all
2
3
4
5
Perfectly
33
Q

A patient is admitted to the intensive care unit (ICU) with a diagnosis of hypertension emergency/crisis. The patient’s blood pressure (BP) is 200/130 mm Hg. The nurse is preparing to administer IV Nitropress (nitroprusside). Upon assessment, which of the following patient findings requires immediate intervention by the nurse?

a) Urine output of 40 cc/mL over the last hour
b) Chest pain score of 3/10 (on a scale of 1 to 10)
c) Left arm numbness and weakness
d) Nausea and severe headache

A

Left arm numbness and weakness

Hypertensive emergencies are acute, life-threatening BP elevations that require prompt treatment in an intensive care setting because of the serious target organ damage that may occur.

The finding of left arm numbness and weakness may indicate the patient is experiencing neurological symptoms associated with an ischemic stroke because of the severely elevated BP and requires immediate interventions.

Aurine output of 40 mL/h is within normal limits. The other findings are likely caused by the hypertension and require intervention, but they do not require action as urgently as the neurologic changes.

How well did you know this?
1
Not at all
2
3
4
5
Perfectly
34
Q

The nurse is caring for a patient prescribed Bumex (bumetanide) for the treatment of stage 2 hypertension. Which of the following indicates the patient is experiencing an adverse effect of the medication?

a) Serum potassium value of 3.0 mEq/L
b) Electrocardiogram (EGG) tracing demonstrating peaked T waves
c) Blood glucose value of 160 mg/dL
d) Urine output of 90 cc/mL 1 hour after medication administration

A

Serum potassium value of 3.0 mEq/L

Bumex is a loop diuretic that can cause fluid and electrolyte imbalances.

Patients taking these medications may experience a low serum potassium level. ECG changes associated with an elevated serum potassium levels include peaked T waves.

Diuresis is a desired effect postadministration of Bumex.

The serum glucose level is elevated and requires intervention; however, this elevation is not associated with the administration of Bumex.

How well did you know this?
1
Not at all
2
3
4
5
Perfectly
35
Q

The nurse understands that an overall goal of hypertension management includes which of the following?

a) The patient maintains a normal blood pressure reading.
b) There is no complaint of postural hypotension.
c) There is no indication of target organ damage.
d) There are no complaints of sexual dysfunction.

A

There is no indication of target organ damage.

Prolonged blood pressure elevation gradually damages blood vessels throughout the body, particularly in target organs such as the heart, kidneys, brain, and eyes.

The overall goal of management is that the patient does not experience target organ damage.

The desired effects of antihypertensives are to maintain a normal BP.

Postural hypotension and sexual dysfunction are side effects of certain antihypertension medications.

How well did you know this?
1
Not at all
2
3
4
5
Perfectly
36
Q

A 55-year-old man newly diagnosed with hypertension returns to his physician’s office for a routine follow-up appointment after several months of treatment with Lopressor (metoprolol). During the nurse’s initial assessment the patient’s blood pressure (BP) is recorded as 180/90 mm Hg. The patient states he does not take his medication as prescribed. The best response by the nurse is which of the following?

a) “Your hypertension must be treated with medications; you need to take your Lopressor every day.”
b) “It is very important for you to take your medication as prescribed, or you could experience a stroke.”
c) “The medication you were prescribed may cause sexual dysfunction; are you experiencing this side effect?”
d) “Be certain to discuss your noncompliance with your medication regimen with the physician.”

A

“The medication you were prescribed may cause sexual dysfunction; are you experiencing this side effect?”

The nurse needs to understand why the patient is not taking his medication.

Lopressor is a beta-blocker. All patients should be informed that beta-blockers might cause sexual dysfunction and that other medications are available if problems with sexual function occur.

The other statements, although true, are nontherapeutic and would not elicit why the patient was not taking his medications as prescribed.

How well did you know this?
1
Not at all
2
3
4
5
Perfectly
37
Q

The nurse is caring for a patient with systolic blood pressure of 135 mm Hg. This finding would be classified as which of the following?

a) Prehypertension
b) Normal
c) Stage 1 hypertension
d) Stage 2 hypertension

A

Prehypertension

A systolic blood pressure of 135 mm Hg is classified as prehypertension. A systolic BP of less than 120 mm Hg is normal. A systolic BP of 140 to 159 mm Hg is stage I hypertension. A systolic BP of greater than or equal to 160 is classified as stage II hypertension.

How well did you know this?
1
Not at all
2
3
4
5
Perfectly
38
Q

The nurse is teaching a patient diagnosed with hypertension about the DASH diet. How many servings of meat, fish, and poultry should a patient consume per day?

a) 7 or 8
b) 4 or 5
c) 2 or 3
d) 2 or fewer

A

2 or fewer

Two or fewer servings of meat, fish, and poultry are recommended in the DASH diet.

How well did you know this?
1
Not at all
2
3
4
5
Perfectly
39
Q

Choose the statements that correctly match the hypertensive medication with its side effect. Select all that apply.

a) With ACE inhibitors, assess for bradycardia.
b) Beta-blockers may cause sedation.
c) With thiazide diuretics, monitor serum potassium levels.
d) With adrenergic inhibitors, cough is a common side effect.
e) Direct vasodilators may cause headache and tachycardia.

A

With thiazide diuretics, monitor serum potassium levels.

Direct vasodilators may cause headache and tachycardia.

Thiazide diuretics may deplete potassium; many clients will need potassium supplementation.

Angiotensin-converting enzyme (ACE) inhibitors can induce a mild to severe dry cough.

Beta-blockers may induce decreased heart rate; pulse rate should be assessed before administration.

Direct vasodilators may cause headache and increased heart rate.

Adrenergic inhibitors can cause sedation and fatigue.

How well did you know this?
1
Not at all
2
3
4
5
Perfectly
40
Q

It is important for the nurse to encourage the patient diagnosed with hypertension to rise slowly from a sitting or lying position for which of the following reasons?

a) Gradual changes in position provide time for the heart to reduce its rate of contraction to resupply oxygen to the brain.
b) Gradual changes in position provide time for the heart to increase rate of contraction to resupply oxygen to the brain.
c) Gradual changes in position help reduce the heart’s work to resupply oxygen to the brain.
d) Gradual changes in position help reduce the blood pressure to resupply oxygen to the brain

A

Gradual changes in position provide time for the heart to increase rate of contraction to resupply oxygen to the brain.

It is important for the nurse to encourage the patient to rise slowly from a sitting or lying position because gradual changes in position provide time for the heart to increase its rate of contraction to resupply oxygen to the brain and not blood pressure or heart rate.

How well did you know this?
1
Not at all
2
3
4
5
Perfectly
41
Q

When measuring the blood pressure in each of the patient’s arms, the nurse recognizes that in the healthy adult, which of the following is true?

a) Pressures may vary, with the higher pressure found in the left arm.
b) Pressures must be equal in both arms.
c) Pressures may vary 10 mm Hg or more between arms.
d) Pressures should not differ more than 5 mm Hg between arms.

A

Pressures should not differ more than 5 mm Hg between arms.

Normally, in the absence of disease of the vasculature, there is a difference of no more than 5 mm Hg between arm pressures.

The pressures in each arm do not have to be equal in order to be considered normal.

Pressures that vary more than 10 mm Hg between arms indicate an abnormal finding. The left arm pressure is not anticipated to be higher than the right as a normal anatomic variant.

How well did you know this?
1
Not at all
2
3
4
5
Perfectly
42
Q

The nurse is caring for a patient newly diagnosed with hypertension. Which of the following statements if made by the patient indicates the need for further teaching?

a) “I think I’m going to sign up for a yoga class twice a week to help reduce my stress.”
b) “When getting up from bed, I will sit for a short period prior to standing up.”
c) “I will consult a dietician to help get my weight under control.”
d) “If I take my blood pressure and it is normal, I don’t have to take my BP pills.”

A

“If I take my blood pressure and it is normal, I don’t have to take my BP pills.”

The patient needs to understand the disease process and how lifestyle changes and medications can control hypertension.

The patient must take his/her medication as directed.

A normal BP indicates the medication is producing its desired effect.

The other responses do not indicate the need for further teaching.

How well did you know this?
1
Not at all
2
3
4
5
Perfectly
43
Q

Hypertension that can be attributed to an underlying cause is termed which of the following?

A

Secondary

Secondary hypertension may be caused by a tumor of the adrenal gland (eg, pheochromocytoma).

Primary hypertension has no known underlying cause.

Essential hypertension has no known underlying cause.

Isolated systolic hypertension is demonstrated by readings in which the systolic pressure exceeds 140 mm Hg and the diastolic measurement is normal or near normal (less than 90 mm Hg).

How well did you know this?
1
Not at all
2
3
4
5
Perfectly
44
Q

Which of the following statements are true when the nurse is measuring blood pressure (BP)? Select all that apply.

a) Using a BP cuff that is too large will give a higher BP measurement.
b) The patient’s arm should be positioned at the level of the heart.
c) Using a BP cuff that is too small will give a higher BP measurement.
d) Ask the patient to sit quietly while the BP is being measured.
e) The patient’s BP should be taken 1 hour after the consumption of alcohol.

A

The patient’s arm should be positioned at the level of the heart.

Using a BP cuff that is too small will give a higher BP measurement.

Ask the patient to sit quietly while the BP is being measured.

These statements are all true when measuring a BP.

When using a BP cuff that is too large the reading will be lower than the actual BP.

The patient should avoid smoking cigarettes or drinking caffeine for 30 minutes before BP is measured.

How well did you know this?
1
Not at all
2
3
4
5
Perfectly
45
Q

The nurse is caring for a client newly diagnosed with secondary hypertension. Which of the following conditions contributes to the development of secondary hypertension?

a) Hepatic function
b) Renal disease
c) Acid-based imbalance
d) Calcium deficit

A

Renal disease

Secondary hypertension occurs when a cause for the high blood pressure can be identified.

These causes include renal parenchymal disease, narrowing of the renal arteries, hyperaldosteronism (mineralocorticoid hypertension), pheochromocytoma, certain medications (e.g., prednisone, epoietin alfa [Epogen]), and coarctation of the aorta.

High blood pressure can also occur with pregnancy; women who experience high blood pressure during pregnancy are at increased risk of ischemic heart disease, heart attacks, strokes, kidney disease, diabetes, and death from heart attack. Calcium deficiency or acid-based imbalance does not contribute to hypertension.

How well did you know this?
1
Not at all
2
3
4
5
Perfectly
46
Q

The nurse is caring for a female client who has had 25 mg of oral hydrochlorothiazide added to her medication regimen for the treatment of hypertension (HTN). Which of the following instructions should the nurse give the patient?

a) “You may drink alcohol while taking this medication.”
b) “Increase the amount of fruits and vegetables you eat.”
c) “Take this medication before going to bed.”
d) “You may develop dry mouth or nasal congestion while on this medication.”

A

“Increase the amount of fruits and vegetables you eat.

Thiazide diuretics cause loss of sodium, potassium, and magnesium.

The patient should be encouraged to eat fruits and vegetables which are high in potassium.

Diuretics cause increased urination; the patient should not take the medication prior to going to bed.

Thiazide diuretics to not cause dry mouth or nasal congestion.

Postural hypotension (side effect) may be potentiated by alcohol.

How well did you know this?
1
Not at all
2
3
4
5
Perfectly
47
Q
  1. Mr. Burke coughs up bloody sputum that Cyrus sends to the laboratory. Bloody sputum is referred to as ____________.
A

Answer: Hemoptysis
Rationale: Hemoptysis is bloody sputum that often accompanies respiratory illnesses such as pneumonia.

How well did you know this?
1
Not at all
2
3
4
5
Perfectly
48
Q

A patient who started smoking in adolescence and continues to smoke 40 years later comes to the clinic. The nurse understands that this patient has an increased risk for being diagnosed with which disorder:

Alcoholism and hypertension
Obesity and diabetes
Stress-related illnesses
Cardiopulmonary disease and lung cancer

A

Cardiopulmonary disease and lung cancer

Effects of nicotine on blood vessels and lung tissue have been proven to increase pathological changes, leading to heart disease and lung cancer.

How well did you know this?
1
Not at all
2
3
4
5
Perfectly
49
Q

A patient has been diagnosed with severe iron deficiency anemia. During physical assessment for which of the following symptoms would the nurse assess to determine the patient’s oxygen status?

Increased breathlessness but increased activity tolerance

Decreased breathlessness and decreased activity tolerance

Increased activity tolerance and decreased breathlessness

Decreased activity tolerance and increased breathlessness

A

Decreased activity tolerance and increased breathlessness

Hypoxia occurs because of decreased circulating blood volume, which leads to decreased oxygen to muscles, causing fatigue, decreased activity tolerance, and a feeling of shortness of breath

How well did you know this?
1
Not at all
2
3
4
5
Perfectly
50
Q

A patient is admitted to the emergency department with suspected carbon monoxide poisoning. Even though the patient’s color is ruddy, not cyanotic, the nurse understands that the patient is at a risk for decreased oxygen-carrying capacity of blood because carbon monoxide does which of the following:

Stimulates hyperventilation, causing respiratory alkalosis

Forms a strong bond with hemoglobin, creating a functional anemia.

Stimulates hypoventilation, causing respiratory acidosis

Causes alveoli to overinflate, leading to atelectasis

A

Forms a strong bond with hemoglobin, creating a functional anemia.

Carbon monoxide strongly binds to hemoglobin, making it unavailable for oxygen binding and transport.

How well did you know this?
1
Not at all
2
3
4
5
Perfectly
51
Q

A 6-year-old boy is admitted to the pediatric unit with chills and a fever of 104°F (40°C). What physiological process explains why the child is at risk for developing dyspnea?

Fever increases metabolic demands, requiring increased oxygen need.
Blood glucose stores are depleted, and the cells do not have energy to use oxygen.
Carbon dioxide production increases as result of hyperventilation.
Carbon dioxide production decreases as a result of hypoventilation.

A

Fever increases metabolic demands, requiring increased oxygen need

When the body cannot meet the increased oxygenation need, the increased metabolic rate causes breakdown of protein and wasting of respiratory muscles, increasing the work of breathing

How well did you know this?
1
Not at all
2
3
4
5
Perfectly
52
Q

A patient is admitted with the diagnosis of severe left-sided heart failure. The nurse expects to auscultate which adventitious lung sounds?

Sonorous wheezes in the left lower lung

Rhonchi midsternum

Crackles only in apex of lungs

Inspiratory crackles in lung bases

A

Decreased effective contraction of left side of heart leads to back up of fluid in the lungs, increasing hydrostatic pressure and causing pulmonary edema, resulting in crackles in lung bases.

How well did you know this?
1
Not at all
2
3
4
5
Perfectly
53
Q

The nurse is caring for a patient who has decreased mobility. Which intervention is a simple and cost-effective method for reducing the risks of stasis of pulmonary secretions and decreased chest wall expansion?

Antibiotics

Frequent change of position

Oxygen humidification

Chest physiotherapy

A

Frequent change of position

Movement not only mobilizes secretions but helps strengthen respiratory muscles by impacting the effectiveness of gas exchange processes.

How well did you know this?
1
Not at all
2
3
4
5
Perfectly
54
Q

A patient is admitted with severe lobar pneumonia. Which of the following assessment findings would indicate that the patient needs airway suctioning?

Coughing up thick sputum only occasionally

Coughing up thin, watery sputum easily after nebulization

Decreased independent ability to cough

Lung sounds clear only after coughing

A

Decreased independent ability to cough

Impaired ability to cough up mucus caused by weakness or very thick secretions indicates a need for suctioning when you know the patient has pneumonia.

How well did you know this?
1
Not at all
2
3
4
5
Perfectly
55
Q

A patient was admitted after a motor vehicle accident with multiple fractured ribs. Respiratory assessment includes signs/symptoms of secondary pneumothorax, which includes which of the following?

Sharp pleuritic pain that worsens on inspiration

Crackles over lung bases of affected lung

Tracheal deviation toward the affected lung

Increased diaphragmatic excursion on side of rib fractures
A

Sharp pleuritic pain that worsens on inspiration

When the lung collapses, the thoracic space fills with air on each inspiration, and the atmospheric air irritates the parietal pleura, causing pain.

How well did you know this?
1
Not at all
2
3
4
5
Perfectly
56
Q

A patient has been newly diagnosed with emphysema. In discussing his condition with the nurse, which of his statements would indicate a need for further education?

“I’ll make sure that I rest between activities so I don’t get so short of breath.”

“I’ll rest for 30 minutes before I eat my meal.”

“If I have trouble breathing at night, I’ll use two to three pillows to prop up.”

“If I get short of breath, I’ll turn up my oxygen level to 6 L/min.”
A

“If I get short of breath, I’ll turn up my oxygen level to 6 L/min.”

Hypoxia is the drive to breathe in a patient with chronic obstructive pulmonary disease who has become used to acidic pH and elevated CO2 levels. Turning up to 6 L/min increases the oxygen level, which turns off the drive to breathe.

How well did you know this?
1
Not at all
2
3
4
5
Perfectly
57
Q

The nurse goes to assess a new patient and finds him lying supine in bed. The patient tells the nurse that he feels short of breath. Which nursing action should the nurse perform first?

Raise the head of the bed to 45 degrees.

Take his oxygen saturation with a pulse oximeter.

Take his blood pressure and respiratory rate.

Notify the health care provider of his shortness of breath.

A

Raise the head of the bed to 45 degrees.

Raising the head of the bed brings the diaphragm down and allows for better chest expansion, thus improving ventilation.

How well did you know this?
1
Not at all
2
3
4
5
Perfectly
58
Q

The nurse is caring for a patient who exhibits labored breathing and uses accessory muscles. The patient has crackles in both lung bases and diminished breath sounds. Which would be priority assessments for the nurse to perform? (Select all that apply.)

SpO2 levels

Amount of sputum production

Change in respiratory rate and pattern

Pain in lower calf area

A

SpO2 levels
Amount of sputum production
Change in respiratory rate and pattern

Pain in the lower calf area indicates vascular, not respiratory, status.

How well did you know this?
1
Not at all
2
3
4
5
Perfectly
59
Q

Which of the following statements made by a student nurse indicates the need for further teaching about suctioning a patient with an endotracheal tube?

“Suctioning the patient requires sterile technique.”

“I’ll apply suction while rotating and withdrawing the suction catheter.”

“I’ll suction the mouth after I suction theendotracheal tube.”

“I’ll instill 5 mL of normal saline into the tube before hyperoxygenating the patient.”

A

“I’ll instill 5 mL of normal saline into the tube before hyperoxygenating the patient.” Corre

Saline has been found to cause more side effects when suctioning and does not increase the amount of secretions removed.

How well did you know this?
1
Not at all
2
3
4
5
Perfectly
60
Q

Two hours after surgery the nurse assesses a patient who had a chest tube inserted during surgery. There is 200 mL of dark-red drainage in the chest tube at this time. What is the appropriate action for the nurse to perform?

Record the amount and continue to monitor drainage

Notify the health care provider

Strip the chest tube starting at the chest

Increase the suction by 10 mm Hg

A

Record the amount and continue to monitor drainage

Dark-red drainage after surgery (50 to 200 mL per hour in first 3 hours) is expected, but be aware of sudden increases greater than 100 mL per hour after the first 3 hours, especially if it becomes bright red in color.

How well did you know this?
1
Not at all
2
3
4
5
Perfectly
61
Q

Which nursing intervention is appropriate for preventing atelectasis in the postoperative patient?

Postural drainage

Chest percussion

Incentive spirometer

Suctioning

A

Incentive spirometer

An incentive spirometer is used to encourage deep breathing to inflate alveoli and open pores of Kohn. The rest are used to treat atelectasis and increased mucus production.

How well did you know this?
1
Not at all
2
3
4
5
Perfectly
62
Q

The nurse needs to apply oxygen to a patient who has a precise oxygen level prescribed. Which of the following oxygen-delivery systems should the nurse select to administer the oxygen to the patient?

Nasal cannula

Venturi mask

Simple face mask without inflated reservoir bag

Plastic face mask with inflated reservoir bag
A

Nasal cannula

A nasal cannula delivers precise, high-flow rates of oxygen.

How well did you know this?
1
Not at all
2
3
4
5
Perfectly
63
Q

Mr. Perry Burke is a 51-year-old African-American patient on the medical-surgical unit for management of chronic bronchitis that has turned into pneumonia.

He works in a paper mill factory where he inhales sawdust and chemicals on a daily basis. Working in an industrial environment causes his lungs to be constantly irritated and inflamed.
Cyrus is the nursing student assigned to Mr. Burke. After reviewing his care plan, the health care provider’s orders, and the nursing notes from the previous shift, Cyrus enters Mr. Burke’s room.

A
How well did you know this?
1
Not at all
2
3
4
5
Perfectly
64
Q
  1. Cyrus finds Mr. Burke restless, agitated, and confused. His pulse is 102 beats/min, and respirations are 42 breaths/min and shallow. He is sitting up in bed grasping the side rails and trying to catch his breath. He is most likely experiencing which of the following conditions?

A. Hyperventilation
B. Hypoventilation
C. Hypoxia
D. Dysrhythmia

A

C. Hypoxia

Rationale: Hypoxia is the decreased diffusion of oxygen from the alveoli to the blood, as in pneumonia.

Signs and symptoms of hypoxia include apprehension, restlessness, inability to concentrate, decreased level of consciousness, dizziness, and behavioral changes.

Vital sign changes include increased pulse rate and rate and depth of respiration.

How well did you know this?
1
Not at all
2
3
4
5
Perfectly
65
Q
  1. Mr. Burke’s condition is causing the clinical sign of shortness of breath. Shortness of breath is referred to as _______________.
A

Dyspnea

Dyspnea is shortness of breath often found in hypoxia

How well did you know this?
1
Not at all
2
3
4
5
Perfectly
66
Q
  1. Mr. Burke’s respiratory rate as determined by Cyrus is 42 breaths/min. This means that he is experiencing apnea.
    A. True
    B. False
A

B. False

Apnea is the absence of breath sounds. Tachypnea is more than 20 breaths/min.

How well did you know this?
1
Not at all
2
3
4
5
Perfectly
67
Q
  1. Mr. Burke coughs up bloody sputum that Cyrus sends to the laboratory. Bloody sputum is referred to as ____________.
A

Hemoptysis

: Hemoptysis is bloody sputum that often accompanies respiratory illnesses such as pneumonia.

How well did you know this?
1
Not at all
2
3
4
5
Perfectly
68
Q

The nurse is analyzing a 6-second electrocardiogram (ECG) tracing. The P waves and QRS complexes are regular. The PR interval is 0.18 seconds long, and the QRS complexes are 0.08 seconds long. The heart rate is calculated at 70 bpm. The nurse correctly identifies this rhythm as which of the following?

a) Sinus tachycardia
b) First-degree atrioventricular (AV) block
c) Junctional tachycardia
d) Normal sinus rhythm

A

Normal sinus rhythm

The ECG tracing shows normal sinus rhythm (NSR). NSR has the following characteristics: ventricular and atrial rate: 60 to 100 beats per minute (bpm) in the adult;

ventricular and atrial rhythm: regular; and QRS shape and duration:

usually normal, but may be regularly abnormal;

P wave: normal and consistent shape, always in front of the QRS;

PR interval: consistent interval between 0.12 and 0.20 seconds and P:QRS ratio: 1:1.

How well did you know this?
1
Not at all
2
3
4
5
Perfectly
69
Q

The nurse is participating in the care of a client requiring emergent defibrillation. The nurse will complete the following steps in which order?

a) Charge the defibrillator to the prescribed voltage.
b) Deliver the prescribed electrical charge.
c) Call “clear” three times ensuring patient and environmental safety.

d) Turn on the defibrillator and place it in “not sync” mode.
e) Apply the multifunction conductor pads to the patient’s chest.

A

This is the sequence of events the nurse should implement when delivering emergent defibrillation. If not followed correctly, the patient and health care team may be placed in danger.

How well did you know this?
1
Not at all
2
3
4
5
Perfectly
70
Q

A nurse is completing a shift assessment on a patient admitted to the telemetry unit with a diagnosis of syncope. The patient’s heart rate is 55 bpm with a blood pressure of 90/66 mm Hg. The patient is also experiencing dizziness and shortness of breath. Which of the following medications will the nurse anticipate administering to the patient based on these clinical findings?

a) Atropine
b) Cardizem
c) Lidocaine
d) Pronestyl

A

Atropine

The patient is demonstrating signs and symptoms of symptomatic sinus bradycardia.

Atropine is the medication of choice in treating symptomatic sinus bradycardia

. Lidocaine treats ventricular dysrhythmias.

Pronestyl treats and prevents atrial and ventricular dysrhythmias.

Cardizem is a calcium channel blocker and treats atrial dysrhythmias

How well did you know this?
1
Not at all
2
3
4
5
Perfectly
71
Q

The nurse is caring for a client who has developed junctional tachycardia with a heart rate (HR) of 80 bpm. Which of the following actions should the nurse complete?

a) Prepare to administer IV lidocaine.
b) Withhold the patient’s oral potassium supplement.
c) Prepare for emergent electrical cardioversion.
d) Request a digoxin level be ordered.

A

Request a digoxin level be ordered.

The nurse should request a digoxin level be obtained. Junctional tachycardia generally does not have any detrimental hemodynamic effect;

it may indicate a serious underlying condition, such as digitalis toxicity, myocardial ischemia, hypokalemia, or chronic obstructive pulmonary disease (COPD).

Potassium supplements do not cause junctional tachycardia.

Lidocaine is indicated for the treatment of premature ventricular contractions (PVCs).

Because junctional tachycardia is caused by increased automaticity, cardioversion is not an effective treatment; in fact, it causes an increase in ventricular rate.

How well did you know this?
1
Not at all
2
3
4
5
Perfectly
72
Q

The nurse is analyzing the electrocardiogram (ECG) tracing of a client newly admitted to the cardiac step-down unit with a diagnosis of chest pain. Which of the following findings indicate the need for follow-up?

a) PR interval that is 0.18 seconds long
b) ST segment that is isoelectric in appearance
c) QT interval that is 0. 46 seconds long
d) QRS complex that is 0.10 seconds long

A

QT interval that is 0. 46 seconds long

The QT interval that is 0.46 seconds long needs to be investigated.

The QT interval is usually 0.32 to 0.40 seconds in duration if the heart rate is 65 to 95 bpm.

If the QT interval becomes prolonged, the patient may be at risk for a lethal ventricular dysrhythmia called torsades de pointes. The other findings are normal.

How well did you know this?
1
Not at all
2
3
4
5
Perfectly
73
Q

A nurse is evaluating a client with a temporary pacemaker. The patient’s ECG tracing shows each P wave followed by the pacing spike. The nurse’s best response is which of the following?

a) Document the findings and continue to monitor the patient.
b) Obtain a 12-lead ECG and a portable chest x-ray.
c) Reposition the extremity and turn the patient to left side.
d) Check the security of all connections and increase the milliamperage.

A

Document the findings and continue to monitor the patient.

Capture is a term used to denote that the appropriate complex is followed by the pacing spike.

In this instance, the patient’s temporary pacemaker is functioning appropriately; all Ps wave followed by an atrial pacing spike.

The nurse should document the findings and continue to monitor the patient.

Repositioning the patient, placing the patient on the left side, checking the security of all connections, and increasing the milliamperage are nursing interventions used when the pacemaker has a loss of capture.

Obtaining a 12-lead ECG and chest x-ray are indicated when there is a loss of pacing-total absence of pacing spikes or when there is a change in pacing QRS shape.

How well did you know this?
1
Not at all
2
3
4
5
Perfectly
74
Q

A 26-year-old male patient, who has been diagnosed with paroxysmal supraventricular tachycardia (PSVT), is being treated in the emergency department. The patient is experiencing occasional runs of PSVT lasting up to several minutes at a time. During these episodes, the patient becomes lightheaded but does not lose consciousness. Which of the following maneuvers may be used to interrupt the patient’s atrioventricular nodal reentry tachycardia (AVNRT)? Select all that apply.

a) Placing the patient’s face in cold water
b) Instructing the patient to vigorously exercise
c) Performing carotid massage.
d) Stimulating the patient’s gag reflex
e) Instructing the patient to breathe deeply

A

Placing the patient’s face in cold water

Performing carotid massage.

Stimulating the patient’s gag reflex

The following vagal maneuvers can be used to interrupt AVNRT:

stimulating the patient’s gag reflex, having the patient hold his breath, cough, bear down, placing his face in cold water, or performing carotid massage.

These measures elicit a vagal response which will slow AV conduction time and help restore a regular rhythm. Because of the risk of a cerebral embolic event, carotid massage is contraindicated in patients with carotid bruits.

If the vagal maneuvers are ineffective, the patient may receive a bolus of adenosine to correct the rhythm; this is nearly 100% effective in terminating AVNRT.

Overexertion and deep inspirations are measures that could precipitate SVT.

How well did you know this?
1
Not at all
2
3
4
5
Perfectly
75
Q

A patient is being treated in the intensive care unit following an acute MI. During the nursing assessment, the patient states shortness of breath and chest pain. In addition, the patient’s blood pressure (BP) is 100/60 mm Hg with a heart rate (HR) of 53 bpm, and the electrocardiogram (ECG) tracing shows more P waves than QRS complexes. Which of the following actions should the nurse complete first?

a) Prepare for defibrillation.
b) Obtain a 12-lead ECG.
c) Initiate transcutaneous pacing.
d) Administer 1 mg of IV atropine.

A

Initiate transcutaneous pacing

The patient is experiencing a third-degree heart block. Transcutaneous pacing should be implemented first.

A permanent pacemaker may be indicated if the block continues.

Defibrillation is not indicated; third-degree heart block does not respond to atropine; a 12-lead ECG may be obtained, but is not completed first

How well did you know this?
1
Not at all
2
3
4
5
Perfectly
76
Q

A patient is admitted to the emergency department (ED) with complaints of chest pain and shortness of breath. The nurse notes an irregular rhythm on the bedside electrocardiograph (ECG) monitor. The nurse counts 9 RR intervals on the patient’s 6-second rhythm tracing. The nurse correctly identifies the patient’s heart rate as which of the following?

a) 90 bpm
b) 100 bpm
c) 70 bpm
d) 80 bpm

A

90 bpm

An alternative but less accurate method for estimating heart rate, which is usually used when the rhythm is irregular, is to count the number of RR intervals in 6 seconds and multiply that number by 10.

The RR intervals are counted, rather than QRS complexes, because a computed heart rate based on the latter might be inaccurately high.

The same methods may be used for determining atrial rate, using the PP interval instead of the RR interval. In this instance, 9 × 10 = 90.

How well did you know this?
1
Not at all
2
3
4
5
Perfectly
77
Q

The nurse is assigned to care for the following patients admitted to a telemetry unit. Which patient should the nurse assess first?

a) A patient who received elective cardioversion 1 hour ago with a heart rate (HR) is 115 bpm
b) A patient whose implantable cardioverter defibrillator (ICD) fired twice on the prior shift requiring amiodarone IV
c) A patient returned from an electrophysiology (EP) procedure 2 hours ago complaining of constipation
d) A patient diagnosed with new onset of atrial fibrillation requiring scheduled IV Cardizem

A

A patient whose implantable cardioverter defibrillator (ICD) fired twice on the prior shift requiring amiodarone IV

The patient’s ICD that has fired on the previous shift should be seen first.

This patient is in need of antidysrhythmic medication and this is the priority intervention. The remaining patients should be seen after this patient and are in no acute distress.

How well did you know this?
1
Not at all
2
3
4
5
Perfectly
78
Q

A patient tells the nurse “my heart is skipping beats again; I’m having palpitations.” After completing a physical assessment, the nurse concludes the patient is experiencing occasional premature atrial complexes (PACs). The nurse should instruct the patient to complete which of the following?

a) Request sublingual nitroglycerin.
b) Lie down and elevate the feet.
c) Apply supplemental oxygen.
d) Avoid caffeinated beverages

A

Avoid caffeinated beverages

If PACs are infrequent, no medical interventions are necessary.

Causes of PACs include caffeine, alcohol, nicotine, stretched atrial myocardium (e.g., as in hypervolemia),

anxiety, hypokalemia (low potassium level),

hypermetabolic states (e.g., with pregnancy),

or atrial ischemia, injury, or infarction. The nurse should instruct the patient to avoid caffeinated beverages.

How well did you know this?
1
Not at all
2
3
4
5
Perfectly
79
Q

A nurse is providing morning care for a patient in the ICU. Suddenly, the bedside monitor shows ventricular fibrillation and the patient becomes unresponsive. After calling for assistance, what action should the nurse take next?

a) Administer intravenous epinephrine.
b) Begin cardiopulmonary resuscitation.
c) Provide electrical cardioversion.
d) Prepare for endotracheal intubation.

A

Begin cardiopulmonary resuscitation.

In the acute care setting, when ventricular fibrillation is noted, the nurse should call for assistance and defibrillate the patient as soon as possible.

If defibrillation is not readily available, CPR is begun until the patient can be defibrillated, followed by advanced cardiovascular life support (ACLS) intervention,

which includes endotracheal intubation and administration of epinephrine.

Electrical cardioversion is not indicated for a patient in ventricular fibrillation.

How well did you know this?
1
Not at all
2
3
4
5
Perfectly
80
Q

The nurse is assigned to care for the following patients admitted to a telemetry unit. Which patient should the nurse assess first?

a) A patient who received elective cardioversion 1 hour ago with a heart rate (HR) is 115 bpm
b) A patient returned from an electrophysiology (EP) procedure 2 hours ago complaining of constipation
c) A patient diagnosed with new onset of atrial fibrillation requiring scheduled IV Cardizem
d) A patient whose implantable cardioverter defibrillator (ICD) fired twice on the prior shift requiring amiodarone IV

A

A patient whose implantable cardioverter defibrillator (ICD) fired twice on the prior shift requiring amiodarone IV

The patient’s ICD that has fired on the previous shift should be seen first. This patient is in need of antidysrhythmic medication and this is the priority intervention. The remaining patients should be seen after this patient and are in no acute distress.

81
Q

The nurse is caring for a 56-year-old male patient who had an implantable cardioverter defibrillator (ICD) implanted 2 days prior. The patient tells the nurse “My wife and I can never have sex again now that I have this ICD.” The nurse’s best response is which of the following?

a) “You really should speak to your wife about your concerns.”
b) “You seem apprehensive about resuming sexual activity.”
c) “I will be sure to share your concerns with the physician.”
d) “Sex is permitted following the implantation of an ICD.”

A

“You seem apprehensive about resuming sexual activity.”

The patient treated with an electronic device experiences not only lifestyle and physical changes but also emotional changes.

At different times during the healing process, the patient may feel angry, depressed, fearful, anxious, or a combination of these emotions.

It is imperative for the nurse to observe the patient’s response to the device and provide the patient and family members with emotional support and teaching as indicated.

Identifying that the patient appears apprehensive about resuming sexual activity acknowledges the patient’s concerns while allowing for further discussion.

The remaining responses ignore the patient’s feelings and do not facilitate an ongoing conversation or explore the patient’s concern.

82
Q

The nurse is caring for a patient following the insertion of a permanent pacemaker. Which of the following discharge instructions are appropriate for the nurse to review with the patient? Select all that apply.

a) Check pulse daily, reporting sudden slowing or increase.
b) Wear a medical alert noting the presence of a pacemaker
c) Avoid the usage of microwave ovens and electronic tools
d) Avoid handheld screening devices in airports.
e) Refrain from walking through antitheft devices.

A

Check pulse daily, reporting sudden slowing or increase.

Wear a medical alert noting the presence of a pacemaker

Avoid handheld screening devices in airports.

Handheld screening devices used in airports may interfere with the pacemaker.

Patients should be advised to ask security personnel to perform a hand search instead of using the handheld screening device.

With a permanent pacemaker, the patient should be instructed initially to restrict activity on the side of implantation.

Patients also should be educated to perform a pulse check daily and to wear or carry medical identification to alert personnel to the presence of the pacemaker.

Patients should walk through antitheft devices quickly and avoid standing in or near these devices. Patients can safely use microwave ovens and electronic tools.

83
Q

A 1-minute ECG tracing of a patient with a regular heart rate reveals 25 small square boxes within an RR interval. The nurse correctly identifies the patient heart rate as which of the following?

a) 80 bpm
b) 100 bpm
c) 60 bpm.
d) 70 bpm

A

60 bpm

A patient’s HR can be obtained from the ECG tracing by several methods.

A 1-minute strip contains 300 large boxes and 1500 small boxes.

Therefore, an easy and accurate method of determining heart rate with a regular rhythm is to count the number of small boxes within an RR interval and divide by 1,500. In this instance, 1,500/25 = 60.

84
Q

A patient admitted to the telemetry unit has a serum potassium level of 6.6 mEq/L. Which of the following electrocardiographic (ECG) characteristics is commonly associated with this laboratory finding?

a) Peaked T waves
b) Occasional U waves
c) Prolonged QT interval
d) Flattened P waves

A

Peaked T waves

The patient’s serum potassium level is high.

The T wave is an ECG characteristic reflecting repolarization of the ventricles. It may become tall or “peaked” if a patient’s serum potassium level is high. The U wave is an ECG waveform characteristic that may reflect Purkinje fiber repolarization.

It is usually seen when a patient’s serum potassium level is low.

The P wave is an ECG characteristic reflecting conduction of an electrical impulse through the atria and is not affected by a patient’s serum potassium level.

The QT interval is an ECG characteristic reflecting the time from ventricular depolarization to repolarization, and is not affected by a patient’s serum potassium level.

85
Q

A patient is being treated in the intensive care unit following an acute MI. During the nursing assessment, the patient states shortness of breath and chest pain. In addition, the patient’s blood pressure (BP) is 100/60 mm Hg with a heart rate (HR) of 53 bpm, and the electrocardiogram (ECG) tracing shows more P waves than QRS complexes. Which of the following actions should the nurse complete first?

a) Prepare for defibrillation.
b) Administer 1 mg of IV atropine.
c) Obtain a 12-lead ECG.
d) Initiate transcutaneous pacing.

A

Initiate transcutaneous pacing.

The patient is experiencing a third-degree heart block. Transcutaneous pacing should be implemented first.

A permanent pacemaker may be indicated if the block continues.

Defibrillation is not indicated; third-degree heart block does not respond to atropine; a 12-lead ECG may be obtained, but is not completed first.

86
Q

A patient with a history of mitral stenosis is admitted to the intensive care unit (ICU) with the abrupt onset of atrial fibrillation. The patient’s heart rate ranges from 120 to 140 bpm. The nurse recognizes that interventions are implemented to prevent the development of which of the following?

a) Renal failure
b) Embolic stroke
c) Heart failure
d) Myocardial infarction

A

Embolic stroke

Intervention is implemented to prevent the development of an embolic event/stroke.

Patients with a history of previous stroke, transient ischemic attack (TIA), embolic event, mitral stenosis, or prosthetic heart valve and who develop atrial fibrillation are at significant risk of developing an embolic stroke.

Antithrombotic therapy is indicated for all patients with atrial fibrillation, especially those at risk of an embolic event, such as a stroke, and is the only therapy that decreases cardiovascular mortality.

These patients are often placed on warfarin, in contrast to patients who have no risk factors, who are often prescribed 81 to 325 mg of aspirin daily.

87
Q

A patient is scheduled for an elective electrical cardioversion for a sustained dysrhythmia lasting for 24 hours. Which of the following interventions is necessary for the nurse to implement prior to the procedure?

a) Administer anticoagulant therapy as prescribed prior to the procedure.
b) Administer the prescribed digitalis to the patient before the scheduled procedure.
c) Maintain the patient on NPO status for 8 hours prior to the procedure.
d) Administer moderate sedation IV and analgesic medication as prescribed.

A

Administer moderate sedation IV and analgesic medication as prescribed.

Before an elective cardioversion, the patient should receive moderate sedation IV as well as an analgesic medication or anesthesia. In contrast, in emergent situations, the patient may not be premedicated.

Digoxin is usually withheld for 48 hours before cardioversion to ensure the resumption of sinus rhythm with normal conduction.

If the cardioversion is elective and the dysrhythmia has lasted longer than 48 hours, anticoagulation performed for a few weeks before cardioversion may be indicated.

The patient is instructed not to eat or drink for at least 4 hours before the procedure.

88
Q

A patient’s ECG tracing reveals a ventricular rate between 250 and 400, with saw-toothed P waves. The nurse correctly identifies this dysrhythmia as which of the following?

a) Ventricular tachycardia
b) Atrial flutter
c) Ventricular fibrillation
d) Atrial fibrillation

A

Atrial flutterThe nurse correctly identifies the ECG tracing as atrial flutter.

Atrial flutter occurs in the atrium and creates impulses at a regular atrial rate between 250 and 400 times per minute.

The P waves are saw-toothed in appearance.

Atrial fibrillation causes a rapid, disorganized, and uncoordinated twitching of atrial musculature.

The atrial rate is 300 to 600, and the ventricular rate is usually 120 to 200 in untreated atrial fibrillation.

There are no discernible P waves. Ventricular fibrillation is a rapid, disorganized ventricular rhythm that causes ineffective quivering of the ventricles.

The ventricular rate is greater than 300 per minute and extremely irregular, without a specific pattern.

The QRS shape and duration is irregular, undulating waves without recognizable QRS complexes. Ventricular tachycardia is defined as three or more PVCs in a row, occurring at a rate exceeding 100 beats per

89
Q

A nurse has provided discharge instructions to a patient who had an implantable cardioverter defibrillator (ICD) implanted. Which of the following statements, made by the patient, indicates the need for further teaching?

a) “I need to take a cardiopulmonary resuscitation (CPR) class now that I have an ICD.”
b) “I will document the date and time if my ICD fires.”
c) “I can play golf with my son in about 2 or 3 weeks.”
d) “I should tell close friends and family members that I have an ICD.”

A

“I need to take a cardiopulmonary resuscitation (CPR) class now that I have an ICD.”

The patient does not need to take a CPR class. However, it is recommended that the family members and friends of a patient who has an ICD learn CPR. The other statements indicate that the nurse’s teaching was effective.

90
Q

A nurse is providing evening care for a patient wearing a continuous telemetry monitor. While the nurse is giving the patient a back rub, the patient’s monitor alarm sounds and the nurse notes a flat line on the bedside monitor system. What is the nurse’s first response?

a) Call a code and obtain the crash cart.
b) Call for assistance and begin CPR.
c) Administer a pericardial thump.
d) Assess the patient and monitor leads.

A

Assess the patient and monitor leads

The nurse should assess the patient and monitor leads first. It is important that the nurse “treat the patient, not the monitor.” Ventricular asystole may often appear on the monitor when leads are displaced. The other interventions are not necessary.

91
Q

The nurse understands that asystole can be caused by several of the following. Select all that apply.

a) Acidosis
b) Hypovolemia
c) Hypothermia
d) Alkalosis
e) Hypoxia

A

Ventricular asystole is treated the same as pulseless electrical activity (PEA),

focusing on high-quality cardiopulmonary resuscitation (CPR)

with minimal interruptions and identifying underlying and contributing factors.

The key to successful treatment is a rapid assessment to identify a possible cause, which is known as the “Hs and Ts”:

hypoxia, hypovolemia, hydrogen ion (acid/base imbalance), hypo- or hyperglycemia, hypo- or hyperkalemia,

hyperthermia, trauma, toxins, tamponade (cardiac), tension pneumothorax, or thrombus (coronary or pulmonary).

92
Q

The nurse is caring for a client who has developed junctional tachycardia with a heart rate (HR) of 80 bpm. Which of the following actions should the nurse complete?

a) Prepare to administer IV lidocaine.
b) Request a digoxin level be ordered.
c) Withhold the patient’s oral potassium supplement.
d) Prepare for emergent electrical cardioversion.

A

Request a digoxin level be ordered

The nurse should request a digoxin level be obtained. Junctional tachycardia generally does not have any detrimental hemodynamic effect;

it may indicate a serious underlying condition, such as digitalis toxicity, myocardial ischemia, hypokalemia, or chronic obstructive pulmonary disease (COPD).

Potassium supplements do not cause junctional tachycardia.

Lidocaine is indicated for the treatment of premature ventricular contractions (PVCs).

Because junctional tachycardia is caused by increased automaticity, cardioversion is not an effective treatment; in fact, it causes an increase in ventricular rate.

93
Q

A 28-year-old female patient presents to the emergency department (ED) stating severe restlessness and anxiety. Upon assessment, the patient’s heart rate is 118 bpm and regular, the patient’s pupils are dilated, and the patient appears excitable. Which action should the nurse take next?

a) Place the patient on supplemental oxygen.
b) Prepare to administer a calcium channel blocker.
c) Instruct the patient to hold her breath and bear down.
d) Question the patient about alcohol and illicit drug use.

A

Question the patient about alcohol and illicit drug use.

The patient is experiencing sinus tachycardia. Since the patient’s findings of tachycardia, dilated pupils, restlessness, anxiety, and excitability can indicate illicit drug use (cocaine),

the nurse should question the patient about alcohol and illicit drug use.

This information will direct the patient’s plan of care.

Causes of tachycardia include medications that stimulate the sympathetic response, stimulants, and illicit drugs.

The treatment goals for sinus tachycardia is usually determined by the severity of symptoms and directed at identifying and abolishing its cause. The other interventions may be implemented, but determining the cause of the tachycardia is essential.

94
Q

The nurse is reviewing the laboratory results for a patient diagnosed with coronary artery disease (CAD). The patient’s low-density lipoprotein (LDL) level is 115 mg/dL. The nurse interprets this value as which of the following?

a) Critically high
b) High
c) Low
d) Within normal limits

A

HighThe normal LDL range is 100 mg/dL to 130 mg/dL. A level of 115 mg/dL is considered to be high. The goal of treatment is to decrease the LDL level below 100 mg/dL (less than 70 mg/dL for very high-risk patients).

95
Q

A patient presents to the emergency room complaining of chest pain. The patient’s orders include the following elements. Which order should the nurse complete first?

a) Oxygen 2 liters nasal cannula
b) Aspirin 325 mg orally
c) 12-lead ECG
d) Troponin level

A

12-lead ECGThe nurse should complete the 12-lead ECG first.

The priority is to determine if the patient is suffering an acute MI and implement appropriate interventions as quickly as possible. The other orders should be completed after the ECG.

96
Q

A nurse is caring for a patient who experienced an MI. The patient is ordered metoprolol (Lopressor). The nurse understands that the therapeutic effect of this medication is which of the following?

a) Decreases resting heart rate
b) Decreases cholesterol level
c) Increases cardiac output
d) Decreases platelet aggregation

A

Decreases resting heart rate

The therapeutic effects of beta-adrenergic blocking agents such as metoprolol are to reduce the myocardial oxygen consumption by blocking beta-adrenergic sympathetic stimulation to the heart.

The result is reduced heart rate, slowed conduction of impulses through the conduction system, decreased blood pressure, and reduced myocardial contractility to balance the myocardial oxygen needs and amount of oxygen available.

This helps to control chest pain and delays the onset of ischemia during work or exercise.

This classification of medication also reduces the incidence of recurrent angina, infarction, and cardiac mortality. Generally the dosage of medication is titrated to achieve a resting heart rate of 50–60 bpm.

Metoprolol is not administered to decrease cholesterol levels, increase cardiac output, or decrease platelet aggregation.

97
Q

In order to be effective, percutaneous transluminal coronary angioplasty (PTCA) must be performed within what time frame, beginning with arrival at the emergency department after diagnosis of myocardial infarction (MI)?

a) 6 to 12 months
b) 60 minutes
c) 9 days
d) 30 minutes

A

60 minutes

The 60-minute interval is known as “door-to-balloon time” for performance of PTCA on a diagnosed MI patient.

The 30-minute interval is known as “door-to-needle time” for administration of thrombolytics post MI.

The time frame of 9 days refers to the time for onset of vasculitis after administration of streptokinase for thrombolysis in an acute MI patient.

The 6- to 12-month time frame refers to the time period during which streptokinase will not be used again in the same patient for acute MI.

98
Q

The nurse understands it is important to promote adequate tissue perfusion following cardiac surgery. Which of the following measures should the nurse complete to prevent deep venous thrombosis (DVT) and possible pulmonary embolism (PE) development? Select all that apply

a) Apply antiembolism stockings.
b) Avoid elevating the knees on the bed.
c) Place pillows in the popliteal space.
d) Encourage the crossing of the legs.
e) Initiate passive exercises.

A

Apply antiembolism stockings.

Avoid elevating the knees on the bed.

Initiate passive exercises.

Preventative measures utilized to prevent venous stasis include:

Application of sequential pneumatic compression wraps or antiembolic stockings; discouraging leg crossing; avoiding elevating the knees on the bed; omitting pillows in the popliteal space;

beginning passive exercises followed by active exercises to promote circulation and prevent venous stasis.

99
Q

The nurse is caring for a patient following a coronary artery bypass graft (CABG). The nurse notes persistent oozing of bloody drainage from various puncture sites. The nurse anticipates that the physician will order which of the following medications to neutralize the unfractionated heparin the patient received?

a) Alteplase (t-PA)
b) Clopidogrel (Plavix)
c) Protamine sulfate
d) Aspirin

A

Protamine sulfate

Protamine sulfate is known as the antagonist for unfractionated heparin (it neutralizes heparin).

Alteplase is a thrombolytic agent.

Clopidogrel (Plavix) is an antiplatelet medication that is given to reduce the risk of thrombus formation post coronary stent placement.

The antiplatelet effect of aspirin does not reverse the effects of heparin.

100
Q

The nurse is caring for a patient newly diagnosed with coronary artery disease (CAD). While developing a teaching plan for the patient to address modifiable risk factors for CAD, the nurse will include which of the following? Select all that apply.

a) Drug use
b) Elevated blood pressure
c) Decreased LDL level
d) Obesity
e) Alcohol use

A

Elevated blood pressure

ObesityHypertension, obesity, hyperlipidemia, tobacco use, diabetes mellitus, metabolic syndrome, and physical inactivity are modifiable risk factor for CAD. Alcohol and drug use are not included in the list of modifiable risk factors for CAD.

101
Q

A nurse is caring for a patient post cardiac surgery. Upon assessment, the patient appears restless and is complaining of nausea and weakness. The patient’s ECG reveals peaked T waves. The nurse reviews the patient’s serum electrolytes anticipating which of the following abnormalities?

a) Hypomagnesemia
b) Hyponatremia
c) Hyperkalemia
d) Hypercalcemia

A

Hyperkalemia

Hyperkalemia is indicated by mental confusion, restlessness, nausea, weakness, and dysrhythmias (tall, peaked T waves).

Hypercalcemia would likely be demonstrated by asystole. Hypomagnesemia would likely be demonstrated by hypotension, lethargy, and vasodilation.

Hyponatremia would likely be indicated by weakness, fatigue, and confusion without change in T-wave formation.

102
Q

Following a percutaneous coronary intervention (PCI), a patient is returned to the nursing unit with large peripheral vascular access sheaths in place. The nurse understands that which of the following methods to induce hemostasis after sheath is contraindicated?

a) Application of a vascular closure device
b) Direct manual pressure
c) Application of a sandbag to the area
d) Application of a mechanical compression device

A

Application of a sandbag to the area

Applying a sandbag to the sheath insertion site is ineffective in reducing the incidence of bleeding and is not an acceptable standard of care.

Application of a vascular closure device (Angioseal, VasoSeal),

direct manual pressure to the sheath introduction site, and application of a mechanical compression device

(C-shaped clamp) are all appropriate methods used to induce hemostasis following peripheral sheath removal.

103
Q

A patient diagnosed with a myocardial infarction (MI) has begun an active rehabilitation program. The nurse recognizes an overall goal of rehabilitation for a patient who has had an MI includes which of the following?

a) Improvement of the quality of life
b) Returning the patient to work and a preillness lifestyle
c) Limiting the effects and progression of atherosclerosis
d) Prevention of another cardiac event

A

Improvement of the quality of life

Overall, cardiac rehabilitation is a complete program dedicated to extending and improving quality of life.

An immediate objective of rehabilitation of the MI patient is to limit the effects and progression of atherosclerosis.

An immediate objective of rehabilitation of the MI patient is to return the patient to work and a preillness lifestyle.

An immediate objective of rehabilitation of the MI patient is to prevent another cardiac event.

104
Q

The nurse is reevaluating a patient 2 hours following a percutaneous transluminal coronary angioplasty (PTCA) procedure. Which of the following assessment findings may indicate the patient is experiencing a complication of the procedure?

a) Heart rate of 100 bpm
b) Potassium level of 4.0 mE/qL
c) Urine output of 40 mL
d) Dried blood at the puncture site

A

Urine output of 40 mL

Complications that may occur following a PTCA include myocardial ischemia, bleeding and hematoma formation, retroperitoneal hematoma, arterial occlusion,

pseudoaneurysm formation, arteriovenous fistula formation, and acute renal failure.

The urine output of 40 mL over a 2-hour period may indicate acute renal failure.

The patient is expected to have a minimum urine output of 30 mL per hour. Dried blood at the insertion site is a finding warranting no acute intervention.

A serum potassium level of 4.0 mEq/L is within normal range.

The heart rate of 100 bmp is within the normal range and indicates no acute distress.

105
Q

The nurse is caring for a patient presenting to the emergency department (ED) complaining of chest pain. Which of the following electrocardiographic (ECG) findings would be most concerning to the nurse?

a) Isolated premature ventricular contractions (PVCs)
b) Frequent premature atrial contractions (PACs)
c) ST elevations
d) Sinus tachycardia

A

ST elevations

The first signs of an acute MI are usually seen in the T wave and ST segment.

The T wave becomes inverted; the ST segment elevates (usually flat).

An elevation in ST segment in two contiguous leads is a key diagnostic indicator for MI (i.e. ST elevation myocardial infarction, STEMI).

This patient requires immediate invasive therapy or fibrinolytic medications.

Although the other ECG findings require intervention, elevated ST elevations require immediate and definitive interventions.

106
Q

The nurse recognizes that the treatment for a non-ST elevation myocardial infarction (NSTEMI) differs from that of a patient with a STEMI, in that a STEMI is more frequently treated with which of the following?

a) IV nitroglycerin
b) Percutaneous coronary intervention (PCI)
c) Thrombolytics
d) IV heparin

A

Percutaneous coronary intervention (PCI)

The patient with a STEMI is often taken directly to the cardiac catheterization laboratory for an immediate PCI. Superior outcomes have been reported with the use of PCI compared to thrombolytics.

IV heparin and IV nitroglycerin are used to treat NSTEMI.

107
Q

A nurse is reevaluating a client receiving IV fibrinolytic therapy. Which of the following patient findings requires immediate intervention by the nurse?

a) Altered level of consciousness
b) Chest pain: 2 of 10 (1-to-10 pain scale)
c) Presence of reperfusion dysrhythmias
d) Minimal oozing of blood from the IV site

A

Altered level of consciousness

A patient receiving fibrinolytic therapy is at risk for complications associated with bleeding.

Altered level of consciousness may indicate hypoxia and intracranial bleeding and the infusion should be discontinued immediately.

Minimal bleeding requires manual pressure.

Reperfusion dysrhythmias are an expected finding.

A chest pain score of 2 is low, and indicates the patient’s chest pain is subsiding, an expected outcome of this therapy.

108
Q

When a patient who has been diagnosed with angina pectoris complains that he is experiencing chest pain more frequently even at rest, the period of pain is longer, and it takes less stress for the pain to occur, the nurse recognizes that the patient is describing which type of angina?

a) Refractory
b) Variant
c) Unstable
d) Intractable

A

Unstable

Unstable angina is also called crescendo or preinfarction angina and indicates the need for a change in treatment.

Intractable or refractory angina produces severe,

incapacitating chest pain that does not respond to conventional treatment.

Variant angina is described as pain at rest with reversible ST-segment elevation and is thought to be caused by coronary artery vasospasm.

Intractable or refractory angina produces severe, incapacitating chest pain that does not respond to conventional treatment.

109
Q

The nurse has completed a teaching session on the self-administration of sublingual nitroglycerin. Which of the following patient statements indicates that the patient teaching has been effective?

a) “After taking two tablets with no relief, I should call emergency medical services.”
b) “I can put the nitroglycerin tablets in my daily pill dispenser with my other medications”.
c) “Side effects of nitroglycerin include, flushing, throbbing headache, and hypertension”.
d) “I can take nitroglycerin prior to having sexual intercourse so I won’t develop chest pain”.

A

“I can take nitroglycerin prior to having sexual intercourse so I won’t develop chest pain”.

Nitroglycerin can be taken in anticipation of any activity that may produce pain.

Because nitroglycerin increases tolerance for exercise and stress when taken prophylactically (i.e. before angina-producing activity, such as exercise, stair-climbing, or sexual intercourse),

it is best taken before pain develops.

The client is instructed to take three tablets 5 minutes apart and if the chest pain is not relieved emergency medical services should be contacted.

Nitroglycerin is very unstable;

it should be carried securely in its original container (e.g., capped dark glass bottle); tablets should never be removed and stored in metal or plastic pillboxes.

Side effects of nitroglycerin includes: flushing, throbbing headache, hypotension, and tachycardia.

110
Q

A patient presents to the ED complaining of anxiety and chest pain after shoveling heavy snow that morning. The patient says that he has not taken nitroglycerin for months but did take three nitroglycerin tablets and although the pain is less, “They did not work all that well.” The patient shows the nurse the nitroglycerin bottle and the prescription was filled 12 months ago. The nurse anticipates which of the following physician orders?

a) Nitroglycerin SL
b) Ativan 1 mg orally
c) Serum electrolytes
d) Chest x-ray

A

Nitroglycerin SL

Nitroglycerin is volatile and is inactivated by heat, moisture, air, light, and time.

Nitroglycerin should be renewed every 6 months to ensure full potency. T

he client’s tablets were expired and the nurse should anticipate administering nitroglycerin to assess if the chest pain subsides.

The other choices may be ordered at a later time, but the priority is to relieve the patient’s chest pain.

111
Q

The nurse is caring for patient experiencing an acute MI (STEMI). The nurse anticipates the physician will prescribe alteplase (Activase). Prior to administering this medication, which of the following questions is most important for the nurse to ask the patient?

a) “What time did your chest pain start today?”
b) “What is your pain level on a scale of 1 to 10?”
c) “Do your parents have a history of heart disease?”
d) “How many sublingual nitroglycerin tabs did you take?”

A

“What time did your chest pain start today?”

The patient may be a candidate for thrombolytic (fibrolytic) therapy.

These medications are administered if the patient’s chest pain lasts longer than 20 minutes, unrelieved by nitroglycerin,

ST-segment elevation in the at least two leads that face the same area of the heart, less than 6 hours from onset of pain.

The most appropriate question for the nurse to ask is in relationship to when the chest pain began.

The other questions would not aid in determining if the patient is a candidate for thrombolytic therapy.

112
Q

A nurse is caring for a patient in the cardiovascular intensive care unit (CVICU) following a coronary artery bypass graft (CABG). Which of the following clinical findings requires immediate intervention by the nurse?

a) Blood pressure: 110/68 mmHg
b) Heart rate: 66 bpm
c) Pain score: 5/10.
d) CVP reading: 1 mmHg

A

CVP reading: 1 mmHg

The central venous pressure (CVP) reading of 1 is low (2–6 mmHg) and indicates reduced right ventricular preload, commonly caused by hypovolemia.

Hypovolemia is the most common cause of decreased cardiac output after cardiac surgery.

Replacement fluids such as colloids, packed red blood cells, or crystalloid solutions may be prescribed.

The other findings require follow-up by the nurse; however, addressing the CVP reading is the nurse’s priority.

113
Q

A patient has had a 12-lead -ECG completed as part of an annual physical examination. The nurse notes an abnormal Q wave on an otherwise unremarkable ECG. The nurse recognizes this finding indicates which of the following?

a) Variant angina
b) A past MI
c) An evolving MI
d) A cardiac dysrhythmia

A

A past MIAn abnormal Q wave may be present without ST-segment and T-wave changes, which indicates an old, not acute, MI.

114
Q

The nurse is caring for a patient diagnosed with unstable angina receiving IV heparin. The patient is placed on bleeding precautions. Bleeding precautions include which of the following measures?

a) Avoiding continuous BP monitoring
b) Using an electric toothbrush
c) Avoiding subcutaneous (SQ) injections
d) Avoiding the use of nail clippers

A

Avoiding continuous BP monitoring

The patient receiving heparin is placed on bleeding precautions,

which can include: applying pressure to the site of any needle punctures for a longer time than usual, avoiding intramuscular injections,

avoiding tissue injury and bruising from trauma or constrictive devices (e.g. continuous use of an automatic BP cuff).

SQ injections are permitted; a soft toothbrush should be used, and the patient may use nail clippers, but with caution.

115
Q

The nurse is caring for a patient who was admitted to the telemetry unit with a diagnosis of rule/out acute MI. The patient’s chest pain began 3 hours ago. Which of the following laboratory tests would be most helpful in confirming the diagnosis of a current MI?

a) Troponin C level
b) Creatinine kinase-myoglobin (CK-MB) level
c) Myoglobin level
d) CK-MM

A

Creatinine kinase-myoglobin (CK-MB) level

Elevated CK-MB assessment by mass assay is an indicator of acute MI; the levels begin to increase within a few hours and peak within 24 hours of an MI.

If the area is reperfused (due to thrombotic therapy or PCI), it peaks earlier. CK-MM (skeletal muscle) is not an indicator of cardiac muscle damage.

There are three isomers of troponin: C, I, and T. Troponin I and T are specific for cardiac muscle, and these biomarkers are currently recognized as reliable and critical markers of myocardial injury.

An increase in myoglobin is not very specific in indicating an acute cardiac event; however, negative results are an excellent parameter for ruling out an acute MI.

116
Q

The nurse is caring for a male patient who is being evaluated for lipid-lowering medication. The patient’s laboratory results reveal the following: Total cholesterol: 230 mg/dL, LDL: 120 mg/dL, and a triglyceride level of 310 mg/dL. Which of the following classes of medications would be most appropriate for the patient based on his laboratory findings?

a) Nicotinic acids
b) HMG-CoA reductase inhibitors
c) Bile acid sequestrants
d) Fibric acids

A

Nicotinic acidsThe most appropriate class of medications based on the patient’s laboratory findings would be nicotinic acids.

This class of medications is prescribed for patients with: minimally elevated cholesterol and LDL levels or as an adjunct to a statin when the lipid goal has not been has not been achieved and triglyceride (TG) levels are elevated.

117
Q

A middle-aged male presents to the ED complaining of severe chest discomfort. Which of the following patient findings is most indicative of a possible MI?

a) Chest discomfort not relieved by rest or nitroglycerin
b) Intermittent nausea and emesis for 3 days
c) Anxiousness, restlessness, and lightheadedness
d) Cool, clammy, diaphoretic, and pale appearance

A

Chest discomfort not relieved by rest or nitroglycerinChest pain or discomfort not relieved by rest or nitroglycerin is associated with an acute MI.

The other findings, although associated with ACS (acute coronary syndrome) or MI, may also occur with angina and, alone, are not indicative of an MI.

118
Q

A patient admitted to the coronary care unit (CCU) diagnosed with a STEMI is anxious and fearful. Which of the following medications will the nurse administer to relieve the patient’s anxiety and decrease cardiac workload?

a) Tenormin (atenolol)
b) IV nitroglycerin
c) IV morphine
d) Norvasc (amlodipine)

A

IV morphine

IV morphine is the analgesic of choice for treatment of an acute MI.

It is given to reduce pain and treat anxiety. It also reduces preload and afterload, which decreases the workload of the heart.

IV nitroglycerin is given to alleviate chest pain. Administration of Tenormin and Norvasc are not indicated in this situation.

119
Q

Which of the following nursing interventions should a nurse perform to reduce cardiac workload in a patient diagnosed with myocarditis?

a) Elevate the patient’s head.
b) Administer supplemental oxygen.
c) Maintain the patient on bed rest.
d) Administer a prescribed antipyretic.

A

Maintain the patient on bed rest

The nurse should maintain the patient on bed rest to reduce cardiac workload and promote healing. Bed rest also helps decrease myocardial damage and the complications of myocarditis.

The nurse should administer supplemental oxygen to relieve tachycardia that may develop from hypoxemia.

If the patient has a fever, the nurse should administer a prescribed antipyretic along with independent nursing measures such as minimizing layers of bed linen,

promoting air circulation and evaporation of perspiration, and offering oral fluids.

The nurse should elevate the patient’s head to promote maximal breathing potential.

120
Q

A patient is admitted to the hospital with possible acute pericarditis and pericardial effusion. The nurse knows to prepare the patient for which diagnostic test used to confirm the patient’s diagnosis?

a) Cardiac cauterization
b) CT scan
c) Echocardiogram
d) Chest x-ray

A

Echocardiogram

Echocardiograms are useful in detecting the presence of the pericardial effusions associated with pericarditis.

An echocardiogram may detect inflammation, pericardial effusion, tamponade, and heart failure. It may help confirm the diagnosis

121
Q

The nurse obtains a health history from a patient with a prosthetic heart valve and new symptoms of infective endocarditis. Which question by the nurse is most appropriate to ask?

a) Do you live with any domesticated animals in your home?
b) Have you recently vacationed outside of the United States?
c) Do you have a family history of endocarditis?
d) Have you been to the dentist recently?

A

Have you been to the dentist recently?

Invasive procedures, particularly those involving mucosal surfaces (e.g., those involving manipulation of gingival tissue or periapical regions of teeth)

can cause a bacteremia, which rarely lasts more than 15 minutes.

However, if a patient has any anatomic cardiac defects or implanted cardiac devices (e.g., prosthetic heart valve, pacemaker,

implantable cardioverter defibrillator [ICD]), bacteremia can cause bacterial endocarditis.

122
Q

A patient is admitted with aortic regurgitation. Which of the following medication classifications are contraindicated since they can cause bradycardia and decrease ventricular contractility?

a) Calcium channel blockers
b) Nitrates
c) Beta blockers
d) Ace inhibitors

A

Calcium channel blockers

The calcium channel blockers diltiazem (Cardizem) and verapamil (Calan, Isoptin) are contraindicated for patients with aortic regurgitation as they decrease ventricular contractility and may cause bradycardia.

123
Q

The nurse is assessing a patient admitted with infective endocarditis. Which of the following manifestations would the nurse expect to find?

a) Small painful lesions on the pads of the fingers and toes
b) Bruising on the palms of the hands and soles of the feet
c) Involuntary muscle movements of the extremities d)Raised red rash on the trunk and face

A

Small painful lesions on the pads of the fingers and toes

Primary presenting symptoms of infective endocarditis are fever and a heart murmur. In addition small, painful nodules (Osler nodes) may be present in pads of fingers or toes.

124
Q

A patient with restrictive cardiomyopathy (RCM) is taking digoxin. Because of the risk of increased sensitivity, the nurse should carefully assess the patient for which of the following manifestations?

a) Anorexia and confusion
b) Tachypnea and dyspnea
c) Abdominal pain and diarrhea
d) Edema and orthopnea

A

Anorexia and confusion

Patients with RCM have increased sensitivity to digoxin, and the nurse must anticipate that low doses will be prescribed and assess for digoxin toxicity.

The most common manifestations of digoxin toxicity are gastrointestinal (anorexia, nausea, and vomiting),

cardiac (rhythm disturbances and heart block), and central nervous system (CNS) disturbances (confusion,

headache, weakness, dizziness, and blurred or yellow vision)

125
Q

A patient who has had a recent myocardial infarction develops pericarditis and complains of level 6 (on a scale of 0–10) chest pain with deep breathing. Which of these ordered pro re nata (PRN) medications will be the most appropriate for the nurse to administer?

a) Fentanyl 2 mg intravenous pyelogram (IVP) every 2–4 hours
b) Ibuprofen (Motrin) 800 mg po every 8 hours
c) Acetaminophen (Tylenol) 650 mg per os (po) every 4 hours
d) Morphine sulfate 6 mg IVP every 2–4 hours

A

Ibuprofen (Motrin) 800 mg po every 8 hours

Pain associated with pericarditis is caused by inflammation, thus nonsteroidal anti-inflammatory drugs (NSAIDs) like ibuprofen are most effective. Opioid analgesics are usually not used for the pain associated with pericarditis.

126
Q

When teaching a patient with rheumatic carditis and a history of recurrent rheumatic fever, which of the following statements made by the patient indicates that teaching has been successful?

a) “I may have to take prophylactic antibiotics for up to 10 years.”
b) “I will avoid any kind of activity.”
c) “I will take nonsteroidal anti-inflammatory medication (NSAIDs) every day.”
d) “I will avoid milk, yogurt and other dairy products.”

A

“I may have to take prophylactic antibiotics for up to 10 years.”

Antibiotic prophylaxis for recurrent rheumatic fever with rheumatic carditis may require 10 or more years of antibiotic coverage (e.g., penicillin G intramuscularly (IM) every 4 weeks, penicillin V orally twice a day (BID),

sulfadiazine orally daily, or erythromycin orally BID.

Patients with a history of rheumatic fever are susceptible to infective endocarditis and should be asked to take prophylactic antibiotics before any invasive procedure, including dental work.

Steroids are prescribed to suppress the inflammatory response and aspirin to control the formation of blood clots around heart valves.

Activities that require minimal activity are recommended to reduce the work of the myocardium and counteract the boredom of weeks of bed rest.

127
Q

A nurse is caring for a patient who had an aortic balloon valvuloplasty. The nurse would inspect the surgical insertion site closely for which of the following complications?

a) Bleeding and wound dehiscence
b) Bleeding and infection
c) Evisceration
d) Thrombosis and infection

A

Bleeding and infectionPossible complications of an aortic balloon valvuloplasty include aortic regurgitation, emboli, ventricular perforation,

rupture of the aortic valve annulus, ventricular dysrhythmia, mitral valve damage, infection, and bleeding from the catheter insertion sites.

128
Q

A nurse reviewing a patient’s echocardiogram report reads the following statements: “The heart muscle is asymmetrically thickened and has an increase in overall size and mass, especially along the septum. The ventricular walls are thickened reducing the size of the ventricular cavities. Several areas of the myocardium have evidence of scaring.” The nurse knows these manifestations are indicative of which type of cardiomyopathy?

a) Dilated
b) Arrhythmogenic right ventricular cardiomyopathy
c) Restrictive
d) Hypertrophi

A

HypertrophicIn hypertrophic cardiomyopathy (HCM), the heart muscle asymmetrically increases in size and mass, especially along the septum. It often affects nonadjacent areas of the ventricle.

The increased thickness of the heart muscle reduces the size of the ventricular cavities and causes the ventricles to take a longer time to relax after systole.

The coronary arteriole walls are also thickened, which decreases the internal diameter of the arterioles. The narrow arterioles restrict the blood supply to the myocardium, causing numerous small areas of ischemia and necrosis.

The necrotic areas of the myocardium ultimately fibrose and scar, further impeding ventricular contraction. Because of the structural changes,

HCM had also been called idiopathic hypertrophic subaortic stenosis (IHSS) or asymmetric septal hypertrophy (ASH).

RCM is characterized by diastolic dysfunction caused by rigid ventricular walls that impair ventricular stretch and diastolic filling.

Arrhythmogenic right ventricular cardiomyopathy (ARVC) occurs when the myocardium of the right ventricle is progressively infiltrated and replaced by fibrous scar and adipose tissue.

129
Q

A nurse is teaching a patient about an upcoming surgery to separate fused cardiac leaflets. Which of the following is the correct term used to describe this surgery?

a) Annuloplasty
b) Valvuloplasty
c) Chordoplasty
d) Commissurotomy

A

CommissurotomyCommissurotomy is the splitting or separating of fused cardiac valve leaflets.

Annuloplasty is a repair of a cardiac valve’s outer ring. Chordoplasty is repair of the stringy, tendinous fibers that connect the free edges of the atrioventricular valve leaflets to the papillary muscle.

Valvuloplasty is a repair of a stenosed or regurgitant cardiac valve by commissurotomy, annuloplasty, leaflet repair, or chordoplasty.

130
Q

Which of the following nursing interventions should a nurse perform to reduce cardiac workload in a patient diagnosed with myocarditis?

a) Administer a prescribed antipyretic.
b) Elevate the patient’s head.
c) Maintain the patient on bed rest.
d) Administer supplemental oxygen.

A

Maintain the patient on bed restThe nurse should maintain the patient on bed rest to reduce cardiac workload and promote healing.

Bed rest also helps decrease myocardial damage and the complications of myocarditis.

The nurse should administer supplemental oxygen to relieve tachycardia that may develop from hypoxemia.

If the patient has a fever, the nurse should administer a prescribed antipyretic along with independent nursing measures such as minimizing layers of bed linen, promoting air circulation and evaporation of perspiration, and offering oral fluids.

The nurse should elevate the patient’s head to promote maximal breathing potential.

131
Q

A nurse is teaching a patient about valve replacement surgery. Which statement by the patient indicates an understanding of the benefit of an autograft replacement valve?

a) “The valve is made from a pig tissue, and I will not need to take any blood-thinning drugs when I am discharged.”
b) “The valve is mechanical, and it will not deteriorate or need replacing.”
c) “The valve is made from my own heart valve, and I will not need to take any blood thinning drugs when I am discharged.”
d) “The valve is from a tissue donor, and I will not need to take any blood thinning drugs with I am discharged.”

A

“The valve is made from my own heart valve, and I will not need to take any blood thinning drugs when I am discharged.

”Autografts (i.e., autologous valves) are obtained by excising the patient’s own pulmonic valve and a portion of the pulmonary artery for use as the aortic valve.

Anticoagulation is unnecessary because the valve is the patient’s own tissue and is not thrombogenic.

The autograft is an alternative for children (it may grow as the child grows), women of childbearing age, young adults, patients with a history of peptic ulcer disease, and people who cannot tolerate anticoagulation.

Aortic valve autografts have remained viable for more than 20 years.

132
Q

The nurse is auscultating the heart of a patient diagnosed with mitral valve prolapse. Which of the following is often the first and only manifestation of mitral valve prolapse?

a) Extra heart sound
b) Syncope
c) Dizziness
d) Fatigue

A

Extra heart soundOften, the first and only sign of mitral valve prolapse is identified when a physical examination of the heart reveals an extra heart sound referred to as a mitral click.

Fatigue, dizziness, and syncope are other symptoms of mitral valve prolapsed.

133
Q

A patient is admitted to the hospital with possible acute pericarditis and pericardial effusion. The nurse knows to prepare the patient for which diagnostic test used to confirm the patient’s diagnosis?

a) Echocardiogram
b) CT scan
c) Cardiac cauterization
d) Chest x-ray

A

EchocardiogramEchocardiograms are useful in detecting the presence of the pericardial effusions associated with pericarditis.

An echocardiogram may detect inflammation, pericardial effusion, tamponade, and heart failure. It may help confirm the diagnosis

134
Q

The nurse is assessing a patient admitted with infective endocarditis. Which of the following manifestations would the nurse expect to find?

a) Small painful lesions on the pads of the fingers and toes
b) Bruising on the palms of the hands and soles of the feet
c) Raised red rash on the trunk and face
d) Involuntary muscle movements of the extremities

A

Small painful lesions on the pads of the fingers and toesPrimary presenting symptoms of infective endocarditis are fever and a heart murmur. In addition small, painful nodules (Osler nodes) may be present in pads of fingers or toes.

135
Q

A patient complaining of heart palpitations is diagnosed with atrial fibrillation caused by mitral valve prolapse. In order to relieve the symptoms, the nurse should teach the patient which of the following dietary interventions?

a) Eliminate caffeine and alcohol
b) A patient complaining of heart palpitations is diagnosed with atrial fibrillation caused by mitral valve prolapse. In order to relieve the symptoms, the nurse should teach the patient which of the following dietary interventions?
c) Decrease the amount of sodium and saturated fat.
d) Decrease the amount of acidic beverages and fruits.

A

Eliminate caffeine and alcohol

To minimize symptoms of mitral valve prolapse, the nurse should instruct the patient to avoid caffeine and alcohol.

The nurse encourages the patient to read product labels, particularly on over-the-counter products such as cough medicine, because these products may contain alcohol, caffeine, ephedrine, and epinephrine, which may produce dysrhythmias and other symptoms.

The nurse also explores possible diet, activity, sleep, and other lifestyle factors that may correlate with symptoms.

136
Q

A nurse caring for a patient with cardiomyopathy determines a diagnosis of anxiety related to a fear of death. Which of the following patient behaviors would indicate to the nurse that the patient’s level of anxiety has decreased?

a) The patient eagerly awaits visits from family.
b) The patient answers questions about physical status with no problem.
c) The patient is resting in bed watching TV.
d) The patient is able to discuss the prognosis freely

A

The patient is able to discuss the prognosis freelyAs anxiety decreases, patients will be able to discuss prognosis freely, verbalize fears and concerns, and participate in support groups.

137
Q

A patient comes into the emergency room complaining about chest pain that gets worse when taking deep breaths and lying down. After ruling out a myocardial infarction, a nurse would assess for which of the following diagnoses?

a) Rheumatic fever
b) Mitral valve stenosis
c) Pericarditis
d) Cardiomyopathy

A

PericarditisThe primary symptom of pericarditis is pain, which is assessed by evaluating the patient in various positions.

The nurse tries to identify whether pain is influenced by respiratory movements while holding an inhaled breath or holding an exhaled breath; by flexion, extension, or rotation of the spine, including the neck;

by movements of shoulders and arms; by coughing; or by swallowing.

Recognizing events that precipitate or intensify pain may help establish a diagnosis and differentiate pain of pericarditis from pain of myocardial infarction.

138
Q

An asymptomatic patient questions the nurse about the diagnosis of mitral regurgitation and inquires about continuing an exercise routine. Which of the following is the most appropriate nursing response?

a) Avoid any type of exercise.
b) Continue the exercise routine but take ample rest after exercising.
c) Avoid strenuous cardiovascular exercise.
d) Continue the exercise routine unless symptoms such as shortness of breath or fatigue develop.

A

Continue the exercise routine unless symptoms such as shortness of breath or fatigue develop

Exercise is not limited until mild symptoms develop.

Once symptoms of heart failure develop, the patient needs to restrict his or her activity level to minimize symptoms.

It is not important for an asymptomatic patient to avoid exercise and to take ample rest after exercise.

139
Q

A patient is admitted with aortic regurgitation. Which of the following medication classifications are contraindicated since they can cause bradycardia and decrease ventricular contractility?

a) Ace inhibitors
b) Beta blockers
c) Calcium channel blockers
d) Nitrates

A

Calcium channel blockers

The calcium channel blockers diltiazem (Cardizem) and verapamil (Calan, Isoptin) are contraindicated for patients with aortic regurgitation as they decrease ventricular contractility and may cause bradycardia.

140
Q

A patient comes to the clinic with complaints of fever, chills, and sore throat and is diagnosed with streptococcal pharyngitis. A nurse knows that early diagnosis and effective treatment is essential to avoid which of the following preventable diseases?

a) Cardiomyopathy
b) Mitral stenosis
c) Rheumatic fever
d) Pericarditis

A

Rheumatic feverRheumatic fever is a preventable disease. Diagnosing and effectively treating streptococcal pharyngitis can prevent rheumatic fever and, therefore, rheumatic heart disease.

141
Q

Which action will a public health nurse include when planning ways to decrease the incidence of rheumatic fever in the community?

a) Provide prophylactic antibiotics to individuals with a family history of rheumatic fever.
b) Teach individuals of the community to seek medical treatment for streptococcal pharyngitis.
c) Encourage susceptible groups in the community to receive immunizations with streptococcal vaccine.
d) Educate individuals of the community about the importance of monitoring temperature when infections occur.

A

Teach individuals of the community to seek medical treatment for streptococcal pharyngitis

Prevention of acute rheumatic fever is dependent upon effective antibiotic treatment of streptococcal pharyngitis.

Family history is not a risk factor for rheumatic fever. There is no immunization that is effective in decreasing the incidence of rheumatic fever.

Education about monitoring temperature will not decrease the incidence of rheumatic fever.

142
Q

When teaching a patient with rheumatic carditis and a history of recurrent rheumatic fever, which of the following statements made by the patient indicates that teaching has been successful?

a) “I will avoid milk, yogurt and other dairy products.”
b) “I may have to take prophylactic antibiotics for up to 10 years.”
c) “I will avoid any kind of activity.”
d) “I will take nonsteroidal anti-inflammatory medication (NSAIDs) every day.”

A

“I may have to take prophylactic antibiotics for up to 10 years.”Antibiotic prophylaxis for recurrent rheumatic fever with rheumatic carditis may require

10 or more years of antibiotic coverage (e.g., penicillin G intramuscularly (IM) every 4 weeks, penicillin V orally twice a day (BID), sulfadiazine orally daily, or erythromycin orally BID.

Patients with a history of rheumatic fever are susceptible to infective endocarditis and should be asked to take prophylactic antibiotics before any invasive procedure, including dental work.

Steroids are prescribed to suppress the inflammatory response and aspirin to control the formation of blood clots around heart valves.

Activities that require minimal activity are recommended to reduce the work of the myocardium and counteract the boredom of weeks of bed rest.

143
Q

A patient with infective endocarditis (IE) and a fever is admitted to the intensive care unit (ICU). Which of these physician orders should the nurse implement first?

a) Administer ceftriaxone (Rocephin) 1 g IVPB q 12 hours.
b) Give acetaminophen (Tylenol) pro re nata (PRN) for fever higher than 100.3 degrees.
c) Order blood cultures drawn from two sites.
d) Obtain a transesophageal echocardiogram.

A

Order blood cultures drawn from two sitesBlood cultures

(with each set including one aerobic and one anaerobic culture) drawn from different venipuncture sites over a 24-hour period

(each set at least 12 hours apart), or every 30 minutes if the patient’s condition is unstable, should be obtained before administration of any antimicrobial agents.

It is essential to obtain blood cultures before initiating antibiotic therapy to obtain accurate sensitivity results.

144
Q

A nurse is teaching a patient who is awaiting a heart transplant. Which of the following statements indicate the patient understands what is required to help minimize rejection?

a) “There is no risk of rejection if the donor heart is an exact match.”
b) “I will need to take three different types of medications for the rest of my life to help prevent rejection.”
c) “I will receive medication before and during surgery which will eliminate the risk of rejection.”
d) “I will need medication following surgery to prevent rejection and if my body does not reject the new heart I will not have to take any medication at home.”

A

“I will need to take three different types of medications for the rest of my life to help prevent rejection.

”Patients who have had heart transplants are constantly balancing the risk of rejection with the risk of infection.

They must adhere to a complex regimen of diet, medications, activity, follow-up laboratory studies, biopsies of the transplanted heart (to diagnose rejection),

and clinic visits.

There are three classes of medications that are prescribed for a transplant patient to help minimize rejection: corticosteroids (e.g., prednisone),

calcineurin inhibitors (tacrolimus, cyclosporin), and antiproliferative agents (mycophenolate mofetil [CellCept], azathioprine [Imuran], or sirolimus [Rapamune]).

145
Q

A patient with restrictive cardiomyopathy (RCM) is taking digoxin. Because of the risk of increased sensitivity, the nurse should carefully assess the patient for which of the following manifestations?

a) Abdominal pain and diarrhea
b) Edema and orthopnea
c) Anorexia and confusion
d) Tachypnea and dyspnea

A

Anorexia and confusionPatients with RCM have increased sensitivity to digoxin, and the nurse must anticipate that low doses will be prescribed and assess for digoxin toxicity.

The most common manifestations of digoxin toxicity are gastrointestinal (anorexia, nausea, and vomiting), cardiac (rhythm disturbances and heart block), and central nervous system (CNS) disturbances (confusion, headache, weakness, dizziness, and blurred or yellow vision).

146
Q

A patient arrives at the ED with an exacerbation of left-sided heart failure and complains of shortness of breath. Which of the following is the priority nursing intervention?

a) Administer angiotensin-converting enzyme inhibitors
b) Assess oxygen saturation level
c) Administer diuretics
d) Administer angiotensin II receptor blockers

A

Assess oxygen saturation levelAssessment is priority to determine severity of the exacerbation.

It is important to assess the oxygen saturation level of a heart failure patient, as below normal oxygen saturation level can be life-threatening.

Treatment options vary according to the severity of the patient’s condition and may include supplemental oxygen, oral and IV medications, major lifestyle changes, implantation of cardiac devices, a

nd surgical approaches. The overall goal of treatment of heart failure is to relieve patient symptoms and reduce the workload on the heart by reducing afterload and preload.

147
Q

Which is a potassium-sparing diuretic used in the treatment of heart failure (HF)?

a) Spironolactone (Aldactone)
b) Chlorothiazide (Diuril)
c) Ethacrynic acid (Edecrin)
d) Bumetanide (Bumex)

A

Spironolactone (Aldactone)Aldactone is a potassium-sparing diuretic. A thiazide diuretic is Diuril. Bumex and Edecrin are loop diuretics.

148
Q

A nurse is teaching patients newly diagnosed with coronary heart disease (CHD) about their disease process and risk factors for heart failure. Which of the following problems can cause left-sided heart failure (HF)?

a) Ineffective right ventricular contraction
b) Myocardial ischemia
c) Cystic fibrosis
d) Pulmonary embolus

A

Myocardial ischemiaMyocardial dysfunction and HF can be caused by a number of conditions including coronary artery disease, hypertension, cardiomyopathy, va

lvular disorders, and renal dysfunction with volume overload.

Atherosclerosis of the coronary arteries is a primary cause of HF, and coronary artery disease is found in the majority of patients with HF.

Ischemia causes myocardial dysfunction because it deprives heart cells of oxygen and causes cellular damage. MI causes focal heart muscle necrosis, the death of myocardial cells,

and a loss of contractility; the extent of the infarction correlates with the severity of HF. Left sided heart failure is caused by myocardial ischemia.

Ineffective right ventricular contraction, pulmonary embolus, and cystic fibrosis cause right-sided heart failure.

149
Q

Which diagnostic study is usually performed to confirm the diagnosis of heart failure?

a) Electrocardiogram (ECG)
b) Blood urea nitrogen (BUN)
c) Serum electrolytes
d) Echocardiogram

A

EchocardiogramAn echocardiogram is usually performed to confirm the diagnosis of heart failure. ECG, serum electrolytes, and a BUN are usually completed in the initial workup.

150
Q

When the nurse observes that the patient has increased difficulty breathing when lying flat, the nurse records that the patient is demonstrating which of the following?

a) Dyspnea on exertion
b) Orthopnea
c) Paroxysmal nocturnal dyspnea
d) Hyperpnea

A

Orthopnea

Patients with orthopnea prefer not to lie flat and will need to maintain their beds in a semi- to high Fowler’s position.

Dyspnea on exertion refers to difficulty breathing with activity.

Hyperpnea refers to increased rate and depth of respiration.

Paroxysmal nocturnal dyspnea refers to orthopnea that occurs only at night.

151
Q

Which of the following is the hallmark of systolic heart failure?

a) Basilar crackles
b) Low ejection fraction (EF)
c) Pulmonary congestion
d) Limitation of activities of daily living (ADLs)

A

Low ejection fraction (EF)

A low EF is a hallmark of systolic heart failure (HF); the severity of HF is frequently classified according to the patient’s symptoms.

152
Q

Which action will the nurse include in the plan of care when caring for a patient admitted with acute decompensated heart failure (ADHF) who is receiving milrinone?

a) Encourage patient to ambulate in room
b) Titrate milrinone rate slowly before discontinuing
c) Teach patient about safe home use of the medication
d) Monitor blood pressure frequently

A

Monitor blood pressure frequently

Milrinone is a phosphodiesterase inhibitor that delays the release of calcium from intracellular reservoirs and prevents the uptake of extracellular calcium by the cells.

This promotes vasodilation, resulting in decreased preload and afterload and reduced cardiac workload. Milrinone is administered intravenously to patients with severe HF, including patients who are waiting for a heart transplant.

Because the drug causes vasodilation, the patient’s blood pressure is monitored prior to administration since if the patient is hypovolemic the blood pressure could drop quickly.

The major side effects are hypotension and increased ventricular dysrhythmias.

Blood pressure and the electrocardiogram (ECG) are monitored closely during and following infusions of milrinone

153
Q

A patient who was admitted to the hospital with a diagnosis of thrombophlebitis 1 day ago suddenly develops complaints of chest pain and shortness of breath and is visibly anxious. The nurse immediately assesses the patient for other signs and symptoms of which of the following problems?

a) Pulmonary embolism
b) Pulmonary edema
c) Myocardial infarction
d) Pneumonia

A

Pulmonary embolismPulmonary embolism is a potentially life-threatening disorder typically caused by blood clots in the lungs.

This disorder poses a particular threat to people with cardiovascular disease.

Blood clots that form in the deep veins of the legs and embolize to the lungs can cause a pulmonary infarction where emboli mechanically obstruct the pulmonary vessels, cutting off the blood supply to sections of the lung.

Clinical indicators of pulmonary embolism can vary but typically include dyspnea, pleuritic chest pain, and tachypnea.

154
Q

Which New York Heart Association classification of heart failure has a poor prognosis and includes symptoms of cardiac insufficiency at rest?

a) I
b) IV
c) III
d) II

A

IV

Symptoms of cardiac insufficiency at rest are classified as IV, according to the New York Heart Association Classification of Heart Failure. In Class I, ordinary activity does not cause undue fatigue, dyspnea, palpitations, or chest pain. In Class II there is a slight limitation of ADLs. In Class III there is marked limitation on ADLs.

155
Q

A nurse is assessing a patient with congestive heart failure for jugular vein distension (JVD). Which of the following observations is important to report to the physician?

a) JVD is noted 1 cm above the sternal angle.
b) JVD is noted at the level of the sternal angle.
c) JVD is noted 3 cm above the sternal angle.
d) No JVD is present.

A

JVD is noted 3 cm above the sternal angleJVD is assessed with the patient sitting at a 45° angle.

Jugular vein distention greater than 3 cm above the sternal angle is considered abnormal and is indicative of right ventricular failure.

156
Q

To assess for peripheral edema, the nurse will examine which of the following areas of the body?

a) Under the sacrum
b) Lips, earlobes
c) Feet, ankles
d) Upper arms

A

Feet, ankles

When right-sided heart failure occurs, blood accumulates in the vessels and backs up in peripheral veins, and the extra fluid enters the tissues.

Particular areas for examination are the dependent parts of the body, such as the feet and ankles.

Other prominent areas prone to edema are the fingers, hands, and over the sacrum.

Cyanosis can be detected by noting color changes in the lips and earlobes.

157
Q

Which of the following describes difficulty breathing when a patient is lying flat?

a) Orthopnea
b) Tachypnea
c) Paroxysmal nocturnal dyspnea (PND)
d) Bradypnea

A

OrthopneaOrthopnea occurs when the patient is having difficulty breathing when lying flat.

Sudden attacks of dyspnea at night are known as paroxysmal nocturnal dyspnea.

Tachypnea is a rapid breathing rate and bradypnea is a slow breathing rate.

158
Q

A patient is prescribed digitalis preparations. Which of the following conditions should the nurse closely monitor when caring for the patient?

a) Vasculitis
b) Potassium levels
c) Enlargement of joints

d)Flexion contractures

A

Potassium levelsA key concern associated with digoxin therapy is digitalis toxicity.

Clinical manifestations of toxicity include anorexia, nausea, visual disturbances, confusion, and bradycardia.

The serum potassium level is monitored because the effect of digoxin is enhanced in the presence of hypokalemia and digoxin toxicity may occur.

159
Q

The nurse identifies which of the following symptoms as a characteristic of right-sided heart failure?

a) Cough
b) Jugular vein distention (JVD)
c) Dyspnea
d) Pulmonary crackles

A

Jugular vein distention (JVD)

JVD is a characteristic of right-sided heart failure. Dyspnea, pulmonary crackles, and cough are manifestations of left-sided heart failure.

160
Q

The nurse identifies which of the following symptoms as a manifestation of right-sided heart failure (HF)?

a) Congestion in the peripheral tissues
b) Accumulation of blood in the lungs
c) Reduction in cardiac output
d) Reduction in forward flow

A

Congestion in the peripheral tissuesRight-sided HF

, failure of the right ventricle, results in congestion in the peripheral tissues and the viscera and causes systemic venous congestion and a reduction in forward flow.

Left-sided HF refers to failure of the left ventricle; it results in pulmonary congestion and causes an accumulation of blood in the lungs and a reduction in forward flow or

cardiac output that results in inadequate arterial blood flow to the tissues.

161
Q

A patient has been diagnosed with congestive heart failure. Which of the following is a cause of crackles heard in the bases of the lungs?

a) Pulmonary hypertension
b) Pulmonary congestion
c) Mitral valve stenosis
d) Heart palpitations

A

Pulmonary congestionCrackles heard in the bases of the lungs are a sign of pulmonary congestion.

Heart palpitations are caused by tachydysrhythmias. Crackles heard in the bases of the lungs are not signs of pulmonary hypertension and mitral valve stenosis.

162
Q

A patient with congestive heart failure is admitted to the hospital with complaints of shortness of breath. How should the nurse position the patient in order to decrease preload?

a) Supine with arms elevated on pillows above the level of the heart
b) Head of the bed elevated at 45 degrees and lower arms supported by pillows
c) Prone with legs elevated on pillows
d) Head of the bed elevated at 30 degrees and legs elevated on pillows

A

Head of the bed elevated at 45 degrees and lower arms supported by pillowsPreload is the amount of blood presented to the ventricle just before systole.

The patient is positioned or taught how to assume a position that facilitates breathing.

The number of pillows may be increased, the head of the bed may be elevated, or the patient may sit in a recliner. In these positions, the venous return to the heart (preload) is reduced, pulmonary congestion is alleviated,

and pressure on the diaphragm is minimized.

The lower arms are supported with pillows to eliminate the fatigue caused by the pull of the patient’s weight on the shoulder muscles.

163
Q

Which of the following nursing interventions should a nurse perform when a patient with valvular disorder of the heart has a heart rate less than 60 beats/min before administering beta blockers?

a) Observe for symptoms of pulmonary edema.
b) Withhold the drug and inform the primary health care provider.
c) Check for signs of toxicity.
d) Continue the drug and document in the patient’s chart.

A

Withhold the drug and inform the primary health care provider.

Before administering beta blockers, the nurse should monitor the patient’s apical pulse. If the heart rate is less than 60 bpm, the nurse should withhold the drug and inform the primary health care provider.

164
Q

The patient with cardiac failure is taught to report which of the following symptoms to the physician or clinic immediately?

a) Ability to sleep through the night
b) Weight loss
c) Persistent cough
d) Increased appetite

A

Persistent cough

Persistent cough may indicate an onset of left-sided heart failure. Loss of appetite should be reported immediately. Weight gain should be reported immediately. Frequent urination, causing interruption of sleep, should be reported immediately.

165
Q

Which of the following medications is categorized as a loop diuretic?

a) Spironolactone (Aldactone)
b) Chlorothiazide (Diuril)
c) Chlorthalidone (Hygroton)
d) Furosemide (Lasix)

A

Furosemide (Lasix)

Lasix is commonly used in the treatment of cardiac failure. Loop diuretics inhibit sodium and chloride reabsorption mainly in the ascending loop of Henle.

Chlorothiazide is categorized as a thiazide diuretic. Chlorthalidone is categorized as a thiazide diuretic.

Spironolactone is categorized as a potassium-sparing diuretic.

166
Q

The nurse understands that a patient with which cardiac arrhythmia is most at risk for developing heart failure?

a) Supraventricular tachycardia
b) Sinus tachycardia
c) First-degree heart block
d) Atrial fibrillation

A

Atrial fibrillationCardiac dysrhythmias such as atrial fibrillation may either cause or result from HF;

in both instances, the altered electrical stimulation impairs myocardial contraction and decreases the overall efficiency of myocardial function.

167
Q

The nurse recognizes which of the following symptoms as a classic sign of cardiogenic shock?

a) High blood pressure
b) Hyperactive bowel sounds
c) Increased urinary output
d) Restlessness and confusion

A

Restlessness and confusion

Cardiogenic shock occurs when decreased cardiac output leads to inadequate tissue perfusion and initiation of the shock syndrome. Inadequate tissue perfusion is manifested as cerebral hypoxia (restlessness, confusion, agitation)

168
Q

A nurse taking care of a patient recently admitted to the ICU observes the patient coughing up large amounts of pink, frothy sputum. Auscultation of the lungs reveals course crackles to lower lobes bilaterally. Based on this assessment, the nurse recognizes this patient is developing which of the following problems?

a)Acute exacerbation of chronic obstructive pulmonary disease

b) Tuberculosis
c) Decompensated heart failure with pulmonary edema
d) Bilateral pneumonia

A

Decompensated heart failure with pulmonary edema

Large quantities of frothy sputum, which is sometimes pink or tan (blood tinged), may be produced, indicating acute decompensated HF with pulmonary edema.

169
Q

A patient arrives at the ED with an exacerbation of left-sided heart failure and complains of shortness of breath. Which of the following is the priority nursing intervention?

a) Assess oxygen saturation level
b) Administer diuretics
c) Administer angiotensin II receptor blockers
d) Administer angiotensin-converting enzyme inhibitors

A

Assess oxygen saturation levelAssessment is priority to determine severity of the exacerbation.

It is important to assess the oxygen saturation level of a heart failure patient, as below normal oxygen saturation level can be life-threatening.

Treatment options vary according to the severity of the patient’s condition and may include supplemental oxygen, oral and IV medications, major lifestyle changes, implantation of cardiac devices,

and surgical approaches. The overall goal of treatment of heart failure is to relieve patient symptoms and reduce the workload on the heart by reducing afterload and preload.

170
Q

The nurse assessing a patient with an exacerbation of heart failure identifies which of the following symptoms as a cerebrovascular manifestation of heart failure (HF)?

a) Tachycardia
b) Dizziness
c) Nocturia
d) Ascites

A

DizzinessCerebrovascular manifestations of heart failure stemming from decreased brain perfusion causes dizziness, lightheadedness, confusion, restlessness, and anxiety due to decreased oxygenation and blood flow

171
Q

The nurse recognizes which of the following lab tests is a key diagnostic indicator of heart failure?

a) Blood urea nitrogen (BUN)
b) Complete blood count (CBC)
c) Creatinine
d) Brain natriuretic peptide (BNP)

A

Brain natriuretic peptide (BNP)The BNP is the key diagnostic indicator of HF. High levels of BNP are a sign of high cardiac filling pressure and can aid in the diagnosis of HF. A BUN, creatinine, and CBC are included in the initial workup.

172
Q

A patient with a diagnosed abdominal aortic aneurysm (AAA) develops severe lower back pain. Which of the following is the most likely cause?

a) The aneurysm has become obstructed.
b) The aneurysm may be preparing to rupture.
c) The patient is experiencing inflammation of the aneurysm.
d) The patient is experiencing normal sensations associated with this condition.

A

)

The aneurysm may be preparing to rupture

Signs of impending rupture include severe back or abdominal pain, which may be persistent or intermittent. Abdominal pain is often localized in the middle or lower abdomen to the left of the midline.

Low back pain may be present because of pressure of the aneurysm on the lumbar nerves.

Indications of a rupturing abdominal aortic aneurysm include constant, intense back pain;

falling blood pressure; and decreasing hematocrit.

Rupture into the peritoneal cavity is rapidly fatal. A retroperitoneal rupture of an aneurysm may result in hematomas in the scrotum, perineum, flank, or penis.

173
Q

The nurse teaches the patient with peripheral vascular disease (PVD) to refrain from smoking because nicotine causes which of the following?

a) Vasospasm
b) Slowed heart rate
c) Depression of the cough reflex
d) Diuresis

A

Vasospasm

Nicotine causes vasospasm and can thereby dramatically reduce circulation to the extremities.

Tobacco smoke also impairs transport and cellular use of oxygen and increases blood viscosity.

Patients with arterial insufficiency who smoke or chew tobacco must be fully informed of the effects of nicotine on circulation and be encouraged to stop.

174
Q

A nurse is completing an assessment on a patient and discovers an enlarged, red, and tender lymph node. The nurse will describe and document the lymph node using which of the following terms?

a) Elephantiasis
b) Lymphangitis
c) Lymphedema
d) Lymphadenitis

A

LymphadenitisAcute lymphadenitis is demonstrated by enlarged, red, and tender lymph nodes.

Lymphangitis is an acute inflammation of the lymphatic channels.

Lymphedema is demonstrated by swelling of tissues in the extremities because of an increased quantity of lymph that results from an obstruction of lymphatic vessels.

Elephantiasis refers to a condition in which chronic swelling of the extremity recedes only slightly with elevation

175
Q

A community health nurse teaches a group of seniors about modifiable risk factors that contribute to the development of peripheral arterial disease (PAD). The nurse knows that the teaching was effective based on which of the following statements?

a) “I will need to stop smoking because the nicotine causes less blood to flow to my hands and feet.”
b) “The older I get the higher my risk for peripheral arterial disease gets.”
c) “Since my family is from Italy, I have a higher risk of developing peripheral arterial disease.”
d) “I will need to increase the amount of green leafy vegetables I eat to lower my cholesterol levels.”

A

“I will need to stop smoking because the nicotine causes less blood to flow to my hands and feet.

”The use of tobacco products may be one of the most important risk factors in the development of atherosclerotic lesions.

Nicotine in tobacco decreases blood flow to the extremities and increases heart rate and blood pressure by stimulating the sympathetic nervous system.

This causes vasoconstriction, thereby decreasing arterial blood flow. It also increases the risk of clot formation by increasing the aggregation of platelets.

176
Q

Which of the following observations regarding ulcer formation on the patient’s lower extremity indicates to the nurse that the ulcer is a result of venous insufficiency?

a) Is very painful to the patient, even though superficial
b) Is very painful to the patient, even though superficial
c) Is deep, involving the joint space
d) Size is large and superficial

A

Size is large and superficialUlcerations are in the area of the medial or lateral malleolus (gaiter area) and are typically large, superficial, and highly exudative.

Superficial venous insufficiency ulcers cause minimal pain. The base of a venous insufficiency ulcer shows beefy red to yellow fibrinous color.

177
Q

In a patient with a bypass graft, the distal outflow vessel must be at least what percentage patent for the graft to remain patent?

a)30

b) 20
c) 50
d) 40

A

50

The distal outflow vessel must be at least 50% patent for the graft to remain patent

178
Q

The nurse completes discharge teaching for a patient following a femoral-to-popliteal bypass graft. What response by the patient would indicate teaching was effective?

a) “I will call if I develop any coldness, numbness, tingling, or pain in the surgical leg.”
b) “I can now stop taking my Lipitor because my leg is fixed.”
c) “I can stop the exercises that were started in the hospital once I return home.”
d) “It will important for me to sit at the kitchen table to promote better breathing.”

A

“I will call if I develop any coldness, numbness, tingling, or pain in the surgical leg.”

The nurse ensures that the patient has the knowledge and ability to assess for any postoperative complications such as infection, occlusion of the artery or graft,

and decreased blood flow. Coldness, numbness, tingling, and pain are signs of peripheral arterial occlusion, and immediate intervention is required.

179
Q

A patient is receiving enoxaparin (Lovenox) and warfarin (Coumadin) therapy for a venous thromboembolism (VTE). Which lab value indicates that anticoagulation is adequate and enoxaparin (Lovenox) can be discontinued?

a) The patient’s prothrombin time (PT) is 0.5 times normal.
b) The patient’s international normalized ratio (INR) is 2.5.
c) The patient’s activated partial thromboplastin time (aPPT) is half of the control value.
d) The patient’s K+ level is 3.5.

A

The patient’s international normalized ratio (INR) is 2.5

Oral anticoagulants, such as warfarin, are monitored by the PT or the INR. Because the full anticoagulant effect of warfarin is delayed for 3 to 5 days,

it is usually administered concurrently with heparin until desired anticoagulation has been achieved (ie, when the PT is 1.5 to 2 times normal or the INR is 2.0 to 3.0)

180
Q

Pentoxifylline (Trental) is a medication used for which of the following conditions?

a) Hypertension
b) Elevated triglycerides
c) Claudication
d) Thromboemboli

A

ClaudicationTrental and Pletal are the only medications specifically indicated for the treatment of claudication.

Thromboemboli, hypertension, and elevated triglycerides are not indications for using Trental

181
Q

The nurse performing an assessment on a patient who has arterial insufficiency of the legs and an ulcer on the left great toe would expect to find which of the following characteristics?

a) Pulses are present, may be difficult to palpate
b) Aching, cramping pain
c) Diminished or absent pulses
d) Superficial ulcer

A

Diminished or absent pulses

Occlusive arterial disease impairs blood flow and can reduce or obliterate palpable pulsations in the extremities. A diminished or absent pulse is a characteristic of arterial insufficiency.

182
Q

The nurse knows which of the following diagnostic tests are used to document the anatomic site of reflux and provides a quantitative measure of the severity of valvular reflux?

a) Duplex ultrasound scan
b) Lymphangiography
c) Contrast phlebography
d) Lymphoscintigraphy

A

Duplex ultrasound scanDiagnostic tests for varicose veins include the duplex ultrasound scan,

which documents the anatomic site of reflux and provides a quantitative measure of the severity of valvular reflux. Contrast phlebography involves injecting a radiopaque contrast agent into the venous system

. Lymphoscintigraphy is done when a radioactively labeled colloid is injected subcutaneously in the second interdigital space.

The extremity is then exercised to facilitate the uptake of the colloid by the lymphatic system, and serial images are obtained at present intervals.

Lymphoangiography provides a way of detecting lymph node involvement resulting from metastatic carcinoma, lymphoma, or infection in sites that are otherwise inaccessible to the examiner except by surgery

183
Q

The term for a diagnostic test that involves injection of a contrast media into the venous system through a dorsal vein in the foot is which of the following?

a) Lymphoscintigraphy
b) Lymphangiography
c) Air plethysmography
d) Contrast phlebography

A

Contrast phlebography

Also known as venography, contrast phlebography involves injecting a radiopaque contrast agent into the venous system.

If a thrombus exists, the x-ray image reveals an unfilled segment of vein in an otherwise completely filled vein.

Air plethysmography quantifies venous reflux and calf muscle pump ejection. In lymphangiography, contrast media are injected into the lymph system.

In a lymphoscintigraphy, a radioactive-labeled colloid is injected into the lymph system.

184
Q

A nurse is teaching a patient newly diagnosed with arterial insufficiency. Which of the following terms should the nurse use to refer to leg pain that occurs when the patient is walking?

a) Orthopnea
b) Intermittent claudication
c) Thromboangiitis obliterans
d) Dyspnea

A

Intermittent claudication

Intermittent claudication is leg pain that is brought on by exercise and relieved by rest.

Dyspnea is the patient’s subjective statement of difficulty breathing. Orthopnea is the inability of the patient to breathe except in the upright (sitting) position.

Thromboangiitis obliterans is a peripheral vascular disease also known as Buerger’s disease.

185
Q

A patient in the emergency department states, “I have always taken a morning walk, but lately my leg cramps and hurts after just a few minutes of walking. The pain goes away after I stop walking, though.” Based on this statement, which priority assessment should the nurse complete?

a) Check for the presence of tortuous veins bilaterally on the legs.
b) Assess for unilateral swelling and tenderness of either leg.
c) Attempt to palpate the dorsalis pedis and posterior tibial pulses.
d) Ask about any skin color changes that occur in response to cold.

A

Attempt to palpate the dorsalis pedis and posterior tibial pulses.

Intermittent claudication is a sign of peripheral arterial insufficiency.

The nurse should assess for other clinical manifestations of peripheral arterial disease in a patient who describes intermittent claudication.

A thorough assessment of the patient’s skin color and temperature and the character of the peripheral pulses are important in the diagnosis of arterial disorders.

186
Q

A patient admitted to the medical surgical unit with a venous thromboembolism (VTE) is started on enoxaparin (Lovenox) and warfarin (Coumadin). The patient asks the nurse why two medications are needed. Which response by the nurse is accurate?

a) “Administration of two anticoagulants decreases the risk of recurrent venous thrombosis.”
b) “Lovenox will dissolve the clot, and Coumadin will prevent any more clots from occurring.”
c) “The Lovenox will work immediately, but the Coumadin takes several days to reach its full effect.”
d) “Because of the potential for a pulmonary embolism, it is important for you to have at least two anticoagulants.”

A

“The Lovenox will work immediately, but the Coumadin takes several days to reach its full effect.”

Oral anticoagulants, such as warfarin, are monitored by the prothrombin time (PT) or the international normalized ratio (INR).

Because the full anticoagulant effect of warfarin is delayed for 3 to 5 days, it is usually administered concurrently with heparin until desired anticoagulation has been achieved (ie, when the PT is 1.5 to 2 times normal or the INR is 2.0 to 3.0).

187
Q

Which of the following statements is accurate regarding Reynaud’s disease?

a) Episodes may be triggered by unusual sensitivity to cold.
b) It is most common in men 16 to 40 years of age.
c) It affects more than two digits on each hand or foot.
d) The disease generally affects the patient bilaterally.

A

Episodes may be triggered by unusual sensitivity to cold.

Episodes of Reynaud’s disease may be triggered by emotional factors or by unusual sensitivity to cold.

The disease is most common in women between 16 and 40 years of age. It is generally unilateral and affects only one or two digits.

188
Q

A patient is receiving enoxaparin (Lovenox) and warfarin (Coumadin) therapy for a venous thromboembolism (VTE). Which lab value indicates that anticoagulation is adequate and enoxaparin (Lovenox) can be discontinued?

a) The patient’s K+ level is 3.5.
b) The patient’s activated partial thromboplastin time (aPPT) is half of the control value.
c) The patient’s international normalized ratio (INR) is 2.5.
d) The patient’s prothrombin time (PT) is 0.5 times normal.

A

The patient’s international normalized ratio (INR) is 2.5.

Oral anticoagulants, such as warfarin, are monitored by the PT or the INR.

Because the full anticoagulant effect of warfarin is delayed for 3 to 5 days, it is usually administered concurrently with heparin until desired anticoagulation has been achieved (ie, when the PT is 1.5 to 2 times normal or the INR is 2.0 to 3.0)

189
Q

The nurse completes discharge teaching for a patient following a femoral-to-popliteal bypass graft. What response by the patient would indicate teaching was effective?

a) “I will call if I develop any coldness, numbness, tingling, or pain in the surgical leg.”
b) “I can now stop taking my Lipitor because my leg is fixed.”
c) “I can stop the exercises that were started in the hospital once I return home.”
d) “It will important for me to sit at the kitchen table to promote better breathing.”

A

“I will call if I develop any coldness, numbness, tingling, or pain in the surgical leg.”

The nurse ensures that the patient has the knowledge and ability to assess for any postoperative complications such as infection, occlusion of the artery or graft, and decreased blood flow.

Coldness, numbness, tingling, and pain are signs of peripheral arterial occlusion, and immediate intervention is required.

190
Q

The nurse teaches the patient with peripheral vascular disease (PVD) to refrain from smoking because nicotine causes which of the following?

a) Vasospasm
b) Depression of the cough reflex
c) Slowed heart rate
d) Diuresis

A

Vasospasm

Nicotine causes vasospasm and can thereby dramatically reduce circulation to the extremities.

Tobacco smoke also impairs transport and cellular use of oxygen and increases blood viscosity.

Patients with arterial insufficiency who smoke or chew tobacco must be fully informed of the effects of nicotine on circulation and be encouraged to stop.

191
Q

The nurse performing an assessment on a patient who has arterial insufficiency of the legs and an ulcer on the left great toe would expect to find which of the following characteristics?

a) Superficial ulcer
b) Aching, cramping pain
c) Diminished or absent pulses
d) Pulses are present, may be difficult to palpate

A

Diminished or absent pulses

Occlusive arterial disease impairs blood flow and can reduce or obliterate palpable pulsations in the extremities. A diminished or absent pulse is a characteristic of arterial insufficiency.

192
Q

A patient admitted to the medical surgical unit with a venous thromboembolism (VTE) is started on enoxaparin (Lovenox) and warfarin (Coumadin). The patient asks the nurse why two medications are needed. Which response by the nurse is accurate?

a) “Because of the potential for a pulmonary embolism, it is important for you to have at least two anticoagulants.”
b) “Administration of two anticoagulants decreases the risk of recurrent venous thrombosis.”
c) “Lovenox will dissolve the clot, and Coumadin will prevent any more clots from occurring.”
d) “The Lovenox will work immediately, but the Coumadin takes several days to reach its full effect.”

A

“The Lovenox will work immediately, but the Coumadin takes several days to reach its full effect.”

Oral anticoagulants, such as warfarin, are monitored by the prothrombin time (PT) or the international normalized ratio (INR). Because the full anticoagulant effect of warfarin is delayed for 3 to 5 days,

it is usually administered concurrently with heparin until desired anticoagulation has been achieved (ie, when the PT is 1.5 to 2 times normal or the INR is 2.0 to 3.0).

193
Q

A nurse is teaching a patient newly diagnosed with arterial insufficiency. Which of the following terms should the nurse use to refer to leg pain that occurs when the patient is walking?

a) Dyspnea
b) Thromboangiitis obliterans
c) Orthopnea
d) Intermittent claudication

A

Intermittent claudication

Intermittent claudication is leg pain that is brought on by exercise and relieved by rest.

Dyspnea is the patient’s subjective statement of difficulty breathing. Orthopnea is the inability of the patient to breathe except in the upright (sitting) position. Thromboangiitis obliterans is a peripheral vascular disease also known as Buerger’s disease.

194
Q

A nurse is completing an assessment on a patient and discovers an enlarged, red, and tender lymph node. The nurse will describe and document the lymph node using which of the following terms?

a) Lymphangitis
b) Lymphedema
c) Elephantiasis
d) Lymphadenitis

A

LymphadenitisAcute lymphadenitis is demonstrated by enlarged, red, and tender lymph nodes.

Lymphangitis is an acute inflammation of the lymphatic channels. Lymphedema is demonstrated by swelling of tissues in the extremities

because of an increased quantity of lymph that results from an obstruction of lymphatic vessels.

Elephantiasis refers to a condition in which chronic swelling of the extremity recedes only slightly with elevation.

195
Q

Which of the following observations regarding ulcer formation on the patient’s lower extremity indicates to the nurse that the ulcer is a result of venous insufficiency?

a) Size is large and superficial
b) Is very painful to the patient, even though superficial
c) Is deep, involving the joint space
d) Is very painful to the patient, even though superficial

A

Size is large and superficial
Explanation:

Ulcerations are in the area of the medial or lateral malleolus (gaiter area) and are typically large, superficial, and highly exudative. Superficial venous insufficiency ulcers cause minimal pain. The base of a venous insufficiency ulcer shows beefy red to yellow fibrinous color.

196
Q

What should the nurse do to manage the persistent swelling in a patient with severe lymphangitis and lymphadenitis?

a) Teach the patient how to apply a graduated compression stocking.
b) Offer cold applications to promote comfort and to enhance circulation
c) Avoid elevating the area.
d) Inform the physician if the temperature remains low.

A

Teach the patient how to apply a graduated compression stocking.
Explanation:

In severe cases of lymphangitis and lymphadenitis with persistent swelling, the nurse teaches the patient how to apply a graduated compression stocking.

The nurse informs the physician if the temperature remains elevated. The nurse recommends elevating the area to reduce the swelling and provides warmth to promote comfort and to enhance circulation.

197
Q

Which of the following are risk factors for venous disorders of the lower extremities?

a) Pacing wires
b) Trauma
c) Obesity
d) Surgery

A

Obesity

Careful assessment is invaluable in detecting early signs of venous disorders of the lower extremities.

Patients with a history of varicose veins, hypercoagulation, neoplastic disease, cardiovascular disease, or recent major surgery or injury are at high risk.

Other patients at high risk include those who are obese or older adults and women taking oral contraceptives.

198
Q

A nurse is changing a dressing on an arterial suture site. The site is red, with foul-smelling drainage. Based on these symptoms, the nurse is aware to monitor for which type of aneurysm?

a) Saccular
b) Dissecting
c) Anastomotic
d) False

A

AnastomoticAn anastomotic aneurysm occurs as a result of infection at arterial suture or graft sites.

Dissection results from a rupture in the intimal layer, resulting in bleeding between the intimal and medial layers of the arterial wall. Saccular aneurysms collect blood in the weakened outpouching.

In a false aneurysm, the mass is actually a pulsating hematoma.

199
Q
A